Вы находитесь на странице: 1из 116

Anatomy Case history: Patient presents with drooping eyelid, visual blurring and bilateral temporal hemianopia (there

was a diagram of the visual field quadrants youre asked to name this abnormality). 1. On a model of thalamus/brainstem identify a. The optic nerve b. Optic chiasm c. Optic tracts d. Ventral tegmental nucleus 2. Unfilled circuit diagram of ventral tegmental nucleus/optic tracts/chiasm/nerves/orbit a. Draw in the tracts b. At which level would the lesion have to be to cause the appearance in this case? c. What could that lesion be? (pituitary adenoma) 3. MRI brain coronal and sagital a. Identify A E (there was an enlarged pituitary) 4. A few more questions on the mechanism of additional features in the case history a. What causes diplopia, cause of drooping eyelid (CN3, ANS fibres to levator palpebrae superioris) b. Where do the ANS fibres run through, what nerve do they travel with? Microbiology 1. Very simple questions from the white book they picked one question from the path visits, one from the lab visits. a. For the autopsy: what happened, cause of death, other findings consistent with/aiding this mechanism of death b. For the lab bit: I wrote FBC + blood film after chemotherapy but I masked the diagnosis so they spent some time asking (easy) stuff about (1) why would you do a blood film in addition to the FBC, what additional information does it provide (2) what was the cellular appearance on blood film (meyloblasts consistent with AML) Case history: UTI 2. 3. 4. 5. 6. 7. 8. Provisional and DDx What tests would you like to do? What would you find on urinalysis/urine microscopy? (> 10^8 bacteria, etc) How is a blood culture performed? What microorganisms cause UTI How would you treat it in the acute hospital setting Side effects of gentamicin, how would you measure them

Pathology Case history: Lung cancer. PHx smoking, malaise, 8 kg weight loss, night sweats, haemoptysis. There was tachypnoea, pleural effusion, temperature 1. 2. 3. 4. 5. Provisional and DDx Investigations for this patient Complications we talked about mets for a bit and How would you investigate the mets (bone scan, head CT, abdo CT, LFTs) Bronchial biopsy revealed a squamous cell carcinoma how would this account for the clinical presentation (tachypnoea, effusion, temperature). 6. How can pneumonia arise in this case? Pharmacology Case history: 42 yo male, BMI 32, nonketotic hyperosmolar coma, first presentation to hospital, BSL (was elevated but not huge) 1. Whats the diagnosis, why? (DKA would not be consistent with the age, BSL) 2. How would you investigate? 3. The patient was prescribed metformin side effects, how is it excreted 4. How would you monitor longterm control of diabetes (BSL, HbA1C) 5. Patient comes back with poor control what pharmacotherapy would you consider next? a. Talked about sulfonylurea would you (1) change to it or (2) add it to metformin. What about in this case? (BMI 32; weight gain) Anatomy Exam Questions 1. Heart anatomy Case History: 55yo male presenting with 2hr hx of central, retrosternal, crushing chest pain and 10 year background of HT, increasing frequency of chest pain over last 2 weeks, chest pain on exertion resolving with rest. He is obese with a BMI of 35. He is tachycardic, tachypnoeic and hypertensive. a) What are the patient's cardiac risk factors (they are highlighted in bold) Middle aged Male Hypertension History of ischemic chest pain Obese a) Provisional and DDx? PDx: Myocardial infarction DDx: cardiac angina, Myocarditis, pericarditis, mitral valve prolapse Aortic aortic dissection, aortic aneurysm Oesophageal oesophagitis, oesophageal spasm, Mallory weiss tears Mediastinal massive PE, tracheitis, malignancy

Musculoskeletal rib fractures, intercostal nerve injury a) What Ix can you do to confirm your provisional DDx? Cardiac enzymes: Troponin T and I proteins that regulate calcium mediated contraction of cardiac mm; peak at 2-4 days, return to normal at 7 days. CK-MB leaks out of fatally injured myocardial cells through damaged cell membranes. Peak 24-36 hours. Return back at 4 days. ECG: T wave peaking within hours and ST elevation. T wave inversion within 24 hours and resolution of ST segment. Within days Q wave formation. a) What single Ix (not bloods) can you do to exclude your differentials? Coronary angiogram: check for stenosis of coronary arteries a) On a model of the heart, point out coronary artery anatomy, describe the course of the coronary arteries, where do they arise from, which arteries drain into coronary sinuses, most likely locations for thrombi to form, which arteries supply the conducting system (SA, AV node) Right coronary aa: originates from the right aortic sinus of the ascending aorta. Passes anteriorly and to the right between the right auricle and the pulmonary trunk and then descends vertically in the coronary sulcus. On reaching the inferior margin of the heart, it turns posteriorly and continues in the sulcus onto the diaphragmatic surface and base of the heart. Left coronary aa: Originates from the left aortic sinus of the ascending aorta. Passes between the pulmonary trunk and the left auricle before entering the coronary sulcus. Divides into two terminal branches, the left anterior descending and circumflex aa. Dominant right: means the posterior IV branch arises from the right coronary artery. Supplies most of the posterior wall of the left ventricle and SA and AV nodes. a) Pathophysiology of MI, how do acute plague changes occur and what happens? Progressive luminal narrowing by plaque enlargement. Hemorrhage into the atheromatous plaque. Thrombus formation initiated by platelet aggregation. Embolization of a thrombus or plaque fragment Coronary artery spasm.

a) Which 3 locations can the heart rupture post MI? What are the consequences of these? What is the problem if the interventricular septum is ruptured? rupture of the left ventricular free wall rupture of the interventricular septum development of mitral regurgitation Septal rupture results in a left-to-right shunt, with right ventricular volume overload, increased pulmonary blood flow, and secondary volume overload of the left atrium and ventricle. As left ventricular systolic function deteriorates and forward flow declines, compensatory vasoconstriction leads to increasing systemic vascular resistance, which, in turn, increases the magnitude of the left-to-right shunt. As the left ventricle fails and the systolic pressure declines, left-to-right shunting decreases and the fraction of the shunt diminishes.

a) What are the common sites for stenosis and atherosclerosis in the coronary circulation? LAD (40-50%) first 2cm, RCA (30-40%) proximal and distal third, LCX (15-20%) first 2cm 1. Epidural haemorrhage Case History: Boy hit on the side of the head a) How does it occur, which part of skull is affected Epidural haemorrhage usually traumatic and associated with fractured skull. Branches of middle meningeal artery are damaged causing haematoma to form between skull and dura. Lucid period of 4-8hours where compensation can occur. Initially get Durets haemorrhages and herniation but with further increase in ICP can get fatal herniation.

b) Name features on head CT

23. Interhemispheric (longitudinal) fissure 24. Head of caudate nucleus 25. Lentiform nucleus (Globus pallidus and Putamen) 26. Thalamus 27. Atrium of lateral ventricle 28. Anterior limb of internal capsule 29. Septum pellucidum 30. Posterior limb of internal capsule c) Glasgow coma scale Glasgow Coma Scale or GCS, is a neurological scale which aims to give a reliable, objective way of recording the conscious state of a person, for initial as well as subsequent assessment. A patient is assessed against the criteria of the scale, and the resulting points give a patient score between 3 (indicating deep unconsciousness) and 15 (fully awake person). Best eye response (4), best verbal response (5) and best motor response (6). Generally, brain injury is classified as: Severe, with GCS 8 - that is also a generally accepted definition of a coma Moderate, GCS 9 - 12 Minor, GCS 13.

1. Cerebrovascular disease Case Protocol 34 sudden onset weakness in left arm and left leg, AF

a) What is the lesion that caused this neurological deficit and what areas are likely to be affected? CVA due to either thrombus superimposed on a preexisting atherosclerotic plaque in an extra/intracranial artery or thromboembolism from the heart secondary to MI, AF or valvular defects. Site: basal ganglia/internal capsule (supplied by lenticulostriate arteries) most likely since no sensory defects, also possible contralateral motor cortex + corona radiata. a) How might this pathological process have occurred? 1. Previous MI 2. Loss of viable myocardial tissue 3. Left ventricular dysfunction 4. Formation of mural thrombus due to blood stasis 5. Embolisation to a cerebral vessel 6. Acute occlusion of a critical BV 7. Cerebral infarct 8. Sudden onset of focal neurological deficits. b) Image of a CT - transverse section - name these parts. What is the function of this part? (caudate) - The caudate, originally thought to primarily be involved with control of voluntary movement, is now known to be an important part of the brain's learning and memory system.

c) Image of a CT - coronal section - which artery supplies these areas? What function is affected if this area was affected (occipital lobe)? What functional areas does the middle cerebral artery supply? occipital lobe: Region in the back of the brain which processes visual information. Not only is the occipital lobe mainly responsible for visual reception, it also contains association areas that help in the visual recognition of shapes and colors. Damage to this lobe can cause visual deficits. Areas supplied by the middle cerebral artery include: The bulk of the lateral surface of the hemisphere; except for the superior inch of the frontal and parietal lobe (anterior cerebral artery), and the inferior part of the temporal lobe. Part of the internal capsule and basal ganglia.

Occlusion of the middle cerebral artery may result in the following defects: 1. Paralysis of the contralateral face and limbs. 2. Sensory loss in the contralateral face and limbs. 3. Aphasia (e.g. Broca's, Wernicke's, conduction, and anomic types) when the dominant hemisphere (usually the left hemisphere for right handed individuals) is affected 4. Contralateral neglect syndrome with damage to the right hemisphere 5. Homonymous hemianopia or quadrantanopia. 1. Case History: Person with past history of hypertension and previous myocardial infarct. Now presents with focal neurological signs and atrial fibrillation. Neurological signs included left spastic hemiparesis of arm and leg, CN 7 affected and upgoing plantar reflex. Identify different parts of brain on CT scans. Blood vessel supply to functional areas of the brain and to identify these areas on CT scan. Territories of ACA/MCA/PCA. What other areas are supplied by the MCA? Anterior cerebral artery Supplies the medial surfaces of the frontal and parietal lobes, frontobasal cortex and corpus collosum. Classically, leg weakness with milder arm involvement, facial sparing. Middle cerebral artery Main trunk occlusion: contralateral hemiplegia and hemianaesthesia, eye deviation towards the infarct, contralateral hemianopia. Neglect visual, motor Autonomic dysfunction Dominant hemisphere: Global aphasia Brocas aphasia (motor) comprehension but impaired expression Wernickes aphasia (sensory) vocalise smoothly, but missing key elements Apraxia inability to perform pre learned tasks Non dominant hemisphere:

Motor, sensory and visual as for dominant Behavioural changes: neglect, dressing apraxia, constructional apraxia, topographic memory deficits, confabulation, general confusion Posterior cerebral artery Supplies the medial inferior temporal lobe, occipital love, occipitoparietal cortex, midbrain, subthalamic nucleus, basal nucleus, and thalamus. Visual field loss Bilateral occipital cortical blindness Unilateral occipital homonymous hemianopia with macular sparing Lat geniculate nucleus hemianopia/quadrantinopia Visual agnosia Lack of recognition/understanding of visual structures Disorders of reading, colour vision and memory MRI sagittal and coronal were also used. Proposed mechanism of death in the person in the case. increased ICP tonsillar herniation through the foramen magnum compression of respiratory/cardiac brainstem centers aspiration pneumonia DVT/PE 1. Spinal injury Case History: 22 yo Surfer dumped into the sand, sustained a neck injury (C6 fracture and anterior transaction of spinal cord) Suspect spinal cord injury if: Head held in unusual position Numbness or tingling that spreads down an arm or leg Weakness Difficulty walking Paralysis (loss of movement) of arms or legs No bladder or bowel control Shock (pale, clammy skin; bluish lips and fingernails; acting dazed or semiconscious) Lack of alertness (unconsciousness) Stiff neck, headache, or neck pain

a) How would you protect the neck? 1. Call 000. 2. Keep the person still. Place heavy towels on both sides of the neck or hold the head and neck to prevent movement. Keep the person in much the same position as he or she was found. 3. Provide as much first aid as possible without moving the person's head or neck. If the person shows no signs of circulation (breathing, coughing or movement), begin CPR, but do not tilt the head back to open the airway. Use your fingers to gently grasp the jaw and lift it forward.

4. If you absolutely must roll the person because he or she is vomiting, choking on blood or in danger of further injury, use at least two people. Work together to keep the person's head, neck and back aligned while rolling the person onto one side. 5. Ideally, the patient is immobilized in the neutral position on a full spine board with a combination of a cervical collar, side head supports, and strapping of the shoulders and pelvis so that the neck is not the center of the body's rotation. Axial CT of spine: What level of the neck is CT taken at? What lesion can you see? What anatomical features are there?

The minimum standard radiographic images taken to evaluate the cervical spine following acute trauma are: a horizontal beam lateral, an AP view, and the open mouth odontoid view. When injuries are serious enough or these views do not adequately demonstrate all diagnostic criteria, the physician may request a CT and/or MRI studies of the cervical spine.

The picture above is a crosstable lateral radiograph of the cervical spine of an unconscious trauma patient. The atlas and axis are disjoined.

The horizontal beam lateral radiograph above demonstrates misalignment of the vertebrae (subluxation) at C5/C6. a) Sagittal MRI of the spine: What injury is present? What complications may result from this lesion (specifically motor and sensory deficits)?

MRI of the cervical spine. Notice the bulges in the discs pressing on the spinal cord between vertebrae C5-C6 and C6-C7. Also note the whitish areas in the spinal cord where its compressed. C3 vertebrae and above : Typically results in loss of diaphragm function, necessitating the use of a ventilator for breathing. C4 : Results in significant loss of function at the biceps and shoulders. C5 : Results in potential loss of function at the shoulders and biceps, and complete loss of function at the wrists and hands. C6 : Results in limited wrist control, and complete loss of hand function.

C7 and T1 : Results in lack of dexterity in the hands and fingers, but allows for limited use of arms. C7 is generally the threshold level for retaining functional independence. Anterior cord syndrome is also an incomplete spinal cord injury. Below the injury, motor function, pain sensation, and temperature sensation is lost; touch, proprioception (sense of position in space), and vibration sense remain intact.

Brown-Squard syndrome usually occurs when the spinal cord is hemisectioned or injured on the lateral side. On the ipsilateral side of the injury (same side), there is a loss of motor function, proprioception, vibration, and light touch. Contralaterally (opposite side of injury), there is a loss of

pain, temperature, and deep touch sensations

a) What short and long term complications may arise from this injury? (bowel/urinary incontinence) SHORT Lung and breathing problems: Injuries at the C-1/C-2 levels will often result in loss of breathing, necessitating mechanical ventilators or phrenic nerve pacing. Bowel and bladder dysfunction infections of the bladderkidney and bladder stones and anal incontinence Sexual dysfunction Autonomic dysreflexia: irritation or pain below the level of the injury sends a signal that fails to reach your brain, producing a reflex action that can constrict blood vessels. The result is a rise in blood pressure and a drop in heart rate that can result in stroke or seizure. Spasticity (increased reflexes and stiffness of the limbs). LONG Neuropathic pain. Atrophy of muscle. DVT and pulmonary embolism Osteoporosis (loss of calcium) and bone degeneration. Pressure sores 1. Prostate anatomy, case protocol 14, 65yo man with thoracic back pain, distended bladder. PSA raised. a) Provisional and DDx (primary and secondary disorders, primary malignancy, trauma, fracture, including secondary malignancies, including prostate, lung, kidney, colon) PDx: metastases to spine from prostate cancer DDx: stroke, TIA, vertebral disc herniation, nerve compression, brain metastases, spinal cord compression, mets from lung cancer b) What investigations are appropriate?(Inc. specifics of how to order the MRI full spine lateral MRI required) FBC: normochromic, normocytic anaemia and thrombocytopenia suggest bone marrow involvement Renal function: urea and creatinine to rule out multiple myeloma Alkaline phosphatase: bone disease or infiltrative liver pathology X ray: spine/chest/abdomen. Osteosclerotic lesions Hypercalcaemia and ALP: bone metastases PSA: raised Bone scan MRI spine sagittal and axial views

c) Look at PSA result and discuss. Discuss public health perspectives of PSA screening. How 'exactly' would you describe risks and benefits to the patient? The PSA test measures the blood level of prostate-specific antigen, an enzyme produced by the prostate. Its normal function is to liquify gelatinous semen after ejaculation, allowing spermatozoa to more easily navigate through the uterine cervix. PSA raised in cancer of the prostate, benign prostatic hypertrophy, infection in the prostate (prostatitis), 24 hours after ejaculation and several days after catheterization. Several other ways of evaluating the PSA have been developed to avoid the shortcomings of simple PSA screening. The use of age-specific reference ranges improves the sensitivity and specificity of the test. The rate of rise of the PSA over time, called the PSA velocity, has been used to evaluate men with PSA levels between 4 and 10 ng/ml, but it has not proven to be an effective screening test. Comparing the PSA level with the size of the prostate, as measured by ultrasound or magnetic resonance imaging, has also been studied. This comparison, called PSA density, is both costly and has not proven to be an effective screening test. d) Discussion of anatomy of prostate (zones)

BPH: the gland is enlarged by nodules arising in the inner (periurethral) portion. Prostate carcinoma: most cases arise in the peripheral part of the prostate, particularly in the posterior region, facilitating palpation during rectal examination. e) How does prostate cancer spread? (intervertebral venous plexus including structure and location) Local invasion: seminal vesicles, urinary bladder, rectum Lymphatic: obturator, perivesical, hypogastric, iliac, presacral, paraortic Haematogenous: bone (lumbar spine, proximal femur, pelvis, thoracic spine, ribs) and viscera (lung, liver) 1. Stomach anatomy, patient with malaena and signs of shock. Ddx Peptic ulcer, gastric cancer, Crohns disease, Oesophageal varices, anti coagulation therapy. Investigations to be done and justify why for each of the test. Obs: BP, O2 sat, HR, temperature Blood cross matching for blood transfusion FBC Hb for anaemia due to blood loss, platelets Coagulation studies anticoagulation therapy? Blood culture and Gram stain Lowenstein Jensen medium and AFB Diagnosis of H.pylori by urea breath test or rapid urease test Endoscopy/Gastroscopy Describe the blood supply to the stomach. Which branches are more likely to be involved in UGI bleed/gastric ulcer. Posterior relations of the stomach. The lesser curvature of the stomach is supplied by the right gastric artery inferiorly, and the left gastric artery superiorly, which also supplies the cardiac region. The greater curvature is supplied by the right gastroepiploic artery inferiorly and the left gastroepiploic artery superiorly.

The fundus of the stomach, and also the upper portion of the greater curvature, are supplied by the short gastric arteries. Posterior relations of stomach Left crus of diaphragm Left suprarenal gland Upper pole of left kidney Splenic artery Anterior pancreatic surface Transverse mesocolon Complications of gastric ulcers. Gastrointestinal bleeding is the most common complication. It occurs when the ulcer erodes one of the blood vessels. Perforation (a hole in the wall) often leads to catastrophic consequences. Erosion of the gastro-intestinal wall by the ulcer leads to spillage of stomach or intestinal content into the abdominal cavity. Perforation at the anterior surface of the stomach leads to acute peritonitis, initially chemical and later bacterial peritonitis. Posterior wall perforation leads to pancreatitis; pain in this situation often radiates to the back. Penetration is when the ulcer continues into adjacent organs such as the liver and pancreas. Scarring and swelling due to ulcers causes narrowing in the duodenum and gastric outlet obstruction. Patient often presents with severe vomiting. Pyloric stenosis Anatomy Exam Questions 1. Heart anatomy Case History: 55yo male presenting with 2hr hx of central, retrosternal, crushing chest pain and 10 year background of HT, increasing frequency of chest pain over last 2 weeks, chest pain on exertion resolving with rest. He is obese with a BMI of 35. He is tachycardic, tachypnoeic and hypertensive. a) What are the patient's cardiac risk factors Male HT New onset of grade III or IV chest pain in the last two weeks Obese b) Provisional and DDx? Acute coronary syndrome, likely to be MI DDx-aortic dissection, tension pneumothorax, oesophagitis, GORD. c) What Ix can you do to confirm your provisional DDx? FBC, EUC, Coags-anaemia, electroylte abnorm?, baseline coags Serum lipids-transient decrease Serial Troponins-elevated

Chest X-ray-APO, HF, exclude other causes of pain ECG-ST elevation, T wave inversion Blood glucose-elevated

d) What single Ix (not bloods) can you do to exclude your differentials? ECG-ST Elevation and T wave inversion (impaired replolarization in the acute infarcted tissue) e) On a model of the heart, point out coronary artery anatomy, describe the course of the coronary arteries, where do they arise from, which arteries drain into coronary sinuses, most likely locations for thrombi to form, which arteries supply the conducting system (SA, AV node) Arteries arise from the aortic sinuses; veins empty into the coronoary sinus which empties into the RA

Cardiac vasculature. A. Anterior view. B. Superior view (atria removed).

A. Anterior view of coronary arterial system. B. Left anterior oblique view of right coronary artery. C. Right anterior oblique view of left coronary artery. Coronary Arteries Right coronary artery Arises in the R aortic sinus Passes anteriorly between R auricle and pulmonary trunk Descends vertically into the coronary sulcus Continues in the suclus post onto the diaphragmatic and base of heart Branches Atrial branch-in groove between R auricle and ascending aorta sinu-atrial nodal branch-passes post around IVC to supply sino-atrial node Right Marginal branch-as RCA approaches inferior surface and continues towards apex Small Branch to the AV node Posterior interventicular branch-final branch and lies in the post interventricular sulcus (R dominant coronary artery) Supplies the RA, RV, sino-atrial node, AV node, interatrial septum, portion of LA, inferpost aspect of interventricular septum, and portion of post aspect of LV. Left coronary arteries Arises in the L aortic sinus Passes between pulmonary trunk and left auricle before entering coronary sulcus

Branches Left anterior descending artery LAD-continues in the anterior interventicular sulcus towards the apex Branches-1 -2 diagonal branches may arise-follow diagonally across the heart Circumflex branch-courses around to the diaphragmatic/base of heart and ends In left dominant coronary artery-continues in the posterior interventicular sulcus Branches-Left marginal artery-continues across rounded obtuse margin of the heart Supplies most of LA, LV and interventicular septum including AV-Bundle

Coronary Veins Coronary sinus receives 4 main veins: Great vein-begins at the apex, ascends in the anterior interventricular groove, continues onto the diaphragmatic surface of the heart in the left coronary sulcus and enlarges to form the coronary sinus Middle cardiac Vein-begins at the apex and ascends in the posterior interventricular groove to fuse with the coronary sinus Small cardiac vein-arises in the lower anterior section of the coronary sulcus between RA and RV, continues into coronary sulcus to the diaphragmatic border of the heart and fuses with coronary sinus at the atrial end May receive right marginal branch; if not this branch empties directly into RA Post Cardiac vein- arises from the Post surface of LV, left of the middle cardiac vein and either enters the coronary sinus directly or joins the great cardiac vein.

T Major cardiac veins. A. Anterior view of major cardiac veins. B. Posteroinferior view

of major cardiac veins. Thrombosis More likely to be proximal Left Main artery 60% of cases

a) Pathophysiology of MI, how do acute plague changes occur and what happens?

b) Which 3 locations can the heart rupture post MI? What are the consequences of these? LV-mortality 90% Interventricular septum-mortality 90% Rupture LV papillary muscle-acute MR leading to CCF c) What is the problem if the interventricular septum is ruptured? Shunting of flow to the RV, leading to Right sided failure. d) What are the common sites for stenosis and atherosclerosis in the coronary circulation? See above

1. Epidural haemorrhage Case History: Boy hit on the side of the head a) How does it occur, which part of skull is affected

Epidural Site Usually due to trauma and resulting fractures leading to rupture of the middle meningeal artery Trauma

Predisposing factors b) Name features on head CT c) Glasgow coma scale

Response Score Eyes Open Spontaneous 4 To Speech 3 To Pain 2 Absent 1 Verbal Converses/Oriented 5 Converses/Desoriented 4 Inapropriate 3 Incomprehensible 2 Absent 1 Motor Obeys 6 Localizes Pain 5 Withdraws(flexion) 4 Decorticate(flexion) Rigidity 3 Decerebrate(extension) Rigidity 2 Absent 1 The sum obtained in this scale is used to the assess Coma and Impaired consciousness Mild is 13 through 15 points Moderate is 9 to 12 points Severe 3 through 8 points Patients with score less than 8 are in Coma

1. Cerebrovascular disease Case Protocol 34 a) What is the lesion that caused this neurological deficit and what areas are likely to be affected? 1. MCA b) How might this pathological process have occurred? Middle cerebral artery Contralateral hemiplegia Eye deviation towards side of infarct

Contralateral hemianopia Contralteral hemianaesthesia Global aphasia (dominant hemisphere) Anosognosia (non-dominant hemisphere) Neglect of contralateral side

c) Image of a CT - transverse section - name these parts. What is the function of this part? (caudate) 1. Function of the caudate-learning and memory, language comprehension and threshold control d) Image of a CT - coronal section - which artery supplies these areas? What function is affected if this area was affected (occipital lobe)? What functional areas does the middle cerebral artery supply?

1. Case History: Person with past history of hypertension and previous myocardial infarct. Now presents with focal neurological signs and atrial fibrillation. Neurological signs included left spastic hemiparesis of arm and leg, CN 7 affected and upgoing plantar reflex. Identify different parts of brain on CT scans. Blood vessel supply to functional areas of the brain and to identify these areas on CT scan. Territories of ACA/MCA/PCA. What other areas are supplied by the MCA? Consequence of PCA occlusion. See notes in CVA module MRI sagittal and coronal were also used. Proposed mechanism of death in the person in the case? Raised intracranial pressure due to the mass from cerebral odema secondary infracted parenchyma Tonsilar herniation (coning) Chest infection Tonsilar herniation (coning) 1. Spinal injury?? Case History: 22 yo Surfer dumped into the sand, sustained a neck injury (C6 fracture and anterior transaction of spinal cord) a) How would you protect the neck? b) Neck brace c) Axial CT of spine: What level of the neck is CT taken at? What lesion can you see? What anatomical features are there?

d) Functional Anatomy
e) The human spine serves to provide structural support and bony protection of the spinal

cord. The cervical spine consists of 7 bony vertebrae separated by flexible intervertebral discs. They are joined together by an intricate network of ligaments, which helps form the normal lordotic curve of the cervical neck.13
f) The spinal column can be divided into 2 separate columns based on function and injury

patterns. The anterior column consists of the bodies of the vertebrae, intervertebral discs, and the anterior and posterior longitudinal ligaments. The function of the vertebral body is to support weight. The posterior column contains the spinal canal and consists of the pedicles, laminae, articulating facets, and transverse and spinous processes. These structures form the vertebral arch, which encloses the vertebral foramen and protects the neural tissues. The arch is formed by bilateral pedicles that are oriented posteriorly and join 2 laminae. The spinous process arises posteriorly from the vertebral arch. The cervical transverse processes and 4 articular processes also arise from the arch. The cervical transverse processes are unique to the vertebral column with an oval foramen transversarium. The vertebral arteries pass through these foramina. The posterior column also includes a group of ligaments including the supraspinous, infraspinous, interspinous, and nuchal ligaments.
g) The first 2 cervical vertebrae are atypical in form and function. The next 5 vertebrae are

all similar in structure and function. The atlas, C1, is a ring-shaped bone that supports the skull. Two concave, superior articular facets articulate with the occipital condyles. The atlas does not have a body or spinous process. The atlas has an anterior and posterior arch, each with a tubercle and lateral mass. The axis, C2, is the strongest cervical vertebrae. The atlas rotates on 2 large articulating surfaces. The odontoid process (dens) projects superiorly from the C2 body and is the bony structure that the atlas rotates on. The odontoid process is held in place by the transverse ligament of the atlas. h)
i) Sagittal MRI of the spine: What injury is present? What complications may result from this lesion (specifically motor and sensory deficits)? j) What short and long term complications may arise from this injury? (bowel/urinary incontinence) 1. Prostate anatomy, case protocol 14, 65yo man with thoracic back pain, distended bladder. PSA raised. a) Provisional and DDx (primary and secondary disorders, primary malignancy, trauma, fracture, including secondary malignancies, including prostate, lung, kidney, colon) b) What investigations are appropriate? c) (Inc. specifics of how to order the MRI - full spine lateral MRI required) Bloods

Urine

FBC, EUC, LFTs-normochromic normocytic anaemia, renal impairement (means poorer prognosis), hypercalcaemia due to bone lysis. Coags-bleeding tendency due to inactivation of plasma procoagulants and reduced platelet func Serum PSA Alkaline phosphatase

Dipstick urinalysis-M/C/S if warrented or suspected infection Imaging Chest X-ray Bone Scan Thoracic CT /full spine lateral MRI required MRI

a) Look at PSA result and discuss. Discuss public health perspectives of PSA screening. How 'exactly' would you describe risks and benefits to the patient? b) No evidence that screening improves mortality. Main screening methods are digital rectal examination and serum PSA. Those likely to be targeted are men >50 with a >10 year survival. c) How does prostate cancer spread? (intervertebral venous plexus including structure and location) d) - mets via inferior vesicle veinsanterior trunk of internal iliac veinsinternal iliac veinscommon illac veinsIVC. e) -other organs include bladder, rectum, renal (chronic renal failure) 1. Stomach anatomy, patient with malaena and signs of shock. (see case protocol 16 notes) Ddx Investigations to be done and justify why for each of the test. Describe the blood supply to the stomach. Which branches are more likely to be involved in UGI bleed/gastric ulcer. Posterior relations of the stomach. Complications of gastric ulcers. 1. Anatomy of the uretur. L sided colicy flank pain classic for renal colic. Provide Ddx and remember AAA You are shown an x- ray KUB with out contrast. Point to where the ureture is, are there any bony landmarks that you can use.

Where is the uretur narrow, and where would you expect to see stones on x ray. a. Stone where the ureter is narrow

At three points along their course the ureters are constricted (Fig. 4.126):

the first point is at the ureteropelvic junction, just inferior to the kidney; the second point is where the ureters cross the common iliac vessels at the pelvic brim; the third point is where the ureters enter the wall of the bladder.

What is the inervation of the uretur, and how does this relate to the way pain is refured in renal colic.

Ureteric vasculature and lymphatics The ureters receive arterial branches from adjacent vessels as they pass towards the bladder (Fig. 4.126):

the renal arteries supply the upper end; the middle part may receive branches from the abdominal aorta, the testicular or ovarian arteries, and the common iliac arteries; in the pelvic cavity, the ureters are supplied by one or more arteries from branches of the internal iliac arteries.

In all cases, arteries reaching the ureters divide into ascending and descending branches, which form longitudinal anastomoses. Lymphatic drainage of the ureters follows a pattern similar to that of the arterial supply. Lymph

from:

the upper part of each ureter drains to the lumbar nodes; the middle part of each ureter drains to lymph vessels; s associated with the common iliac

the inferior part of each ureter drains to lymph nodes associated with the external and internal iliac vessels.

Ureteric innervation

Ureteric innervation is from the renal, aortic, superior hypogastric, and inferior hypogastric plexuses through nerves that follow the blood vessels.

Visceral efferent fibers come from both sympathetic and parasympathetic sources, while visceral afferent fibers return to T11 to L2 spinal cord levels. Ureteric pain, which is usually related to distension of the ureter, is therefore referred to cutaneous areas supplied by T11 to L2 spinal cord levels. These areas would most likely include the posterior and lateral abdominal wall below the ribs and above the iliac crest, the pubic region, the scrotum in males, the labia majora in females, and the proximal anterior aspect of the thigh.

Anatomy 53 yo postmenopausal woman, not on HRT. Noticed some vaginal bleeding, abdo discomfort, urinary symptoms (frequency and urgency from what I can remember). Exam reveals mild ascites, bimanual palp felt a nodular mass in left inguinal region. Adductor weakness on the left side Q: differential diagnosis, investigations (what could be seen on US?) Q: What does the weakness indicate? (obturator nerve involvement) Q: What other structures can be involved in ovarian cancer? How does it spread? (peritoneum, bladder, uterus, rectum, left ureter hydronephrosis) Micro/lab visits

Asked about the nephrectomy I saw and the lab tests, blood smear, antibiotics sensitivity

Case about a 30 yo male IVDU with symptoms of IE. What organisms cause IE? What investigations? Asked about collection of blood culture and sputum collection. What makes a good sputum sample?

Path

Really straightforward case of a 13yo with first presentation of DKA, hypovolaemic shock. Interpret ABG, UEC (whats the significance of apparent hyperkalaemia? Significance of elevated urea, creatinine) Long term glycaemic control (selfmonitoring, HBa1c).

Pharmacology

Women with moderate obesity and DM undergone surgery. Developed leg swelling not related to trauma.

Q: differential diagnosis, investigation Q: How to commence anticoagulation (heparin for first 5 days when starting warfarin) Q: pharmacology of warfarin and heparin. Difference between heparin and lowmolecular weight heparin. Q: Bleeding risk with heparin, how to monitor? How to monitor warfarin? New drugs to switch the pt to in case of heparin-induced thrombocytopenia (have no idea haha sorry).
BIOMED Questions from the Path Exam 1. White book station: UTI: describing what are the diagnostic criteria, defining haematocrit; pathology of gastric ulcers vs duodenal ulcers and complications 2. Anatomy: anatomy of larynx, risk factors for laryngeal cancer, drainage of head and neck cancers 3. Pharmacology: cardiovascular risks, using the NZ Heart Risk Assessment, what medications to use, how aspirin works 4. Path: Breast lump in a young female, fibroadenoma

Questions from Path Exam 1. Interstitial nephritis 2. Anatomy: lymph drainage of the pelvic organs, identifying ureters as possible sites of trauma Tessa Either way here are my biomed q's from last year. On the whole they were very predictable (and then in the next group after me they slammed them with random shit, including anatomy and lymphatic drainage of the larynx): 1. Stroke syndrome, brain anatomy incl vascular supply, MRI scan, "where are the centres for personality?" --> wtf random. I assumed they were getting at frontal damage... but asking it obtusely. 2. Anticoagulation in a post partum woman after PE. 3. Interpreting renal function tests and ABG. Very little interest in white book: what are the kinds of lung cancer that your lung lump could have been? 4. Pathology of IBD. Features, management, prognosis, blah. So straightforward Kumar was basically asleep.

Emmy1. How does aspirin work 2 Breast cancer pathology 3. how to determine mid stream urine 4. Laryngeal cancer anatomy. Model of trachea. Risk factors. Epithelium of larynx. 5. Pharm - cardio risk factors.

Anatomy - Gynae Anatomy with model - asked about types of hysterectomies, which artery is this (uterine), what lymph nodes does uterus, cervix, vagina drain to (Wikipedia has stuff on this) Pathology Person comes in with stroke what are risk factors (i.e. ischaemic vs haemorrhagic, atherosclerotic vs cardiac causes) what are some physical examination findings you may find with people with risk factors (e.g. bruits, AAA, PVD, diabetes, HTN etc), then processes that lead to stroke (atherosclerosis, thrombosis, occlusion etc) Investigations asked about my ABx sensitivity, then questions on DKA (interpret ABG (acidosis with respiratory compensation), why do you get DKA Pharm Patient presents with cough, fever ask for differentials, investigations which organisms (typical Strep Pneumon, or Atypical mycoplasm), How would

you test for mycoplasm (serology, not culture typically) what Abx would you use for mycoplasm and why (macrolides).

Coffs: (we're not exactly sure of questions)

Anatomy - Case history was about renal cell carcinoma, on CT identify - descending aorta, IVC, renal artery, renal vein

Pharmacology - antihypertensives, mechanisms of actions, side-effects, contraindications 1. Loop diuretic eg. frusemide Block the Na-K-2Cl symporter in the ascending loop of Henle, which is responsible for reabsorbing 25% of filtered Na.

Common ADRs include: hypernatremia, hypokalemia, hypomagnesemia, dehydration, hyperuricemia, gout, dizziness, postural hypotension, syncope

2. Thiazide diuretic eg. hydrochlorothiazide Act on the early DCT to block the Na-Cl transporter. Only 5% of filtered sodium load reabsorbed here.

Side effects include hypokalemia, increased serum cholesterol, triglyceride, impaired glucose tolerance, diabetes mellitus and impotence 3. Potassium sparing diuretics Hypokalemia is a problem associated with the use of thiazide or loop diuretics. Signs and symptoms of hypokalemia include muscle weakness, drowsiness, CNS manifestations and cardiac rhythm disturbances. Na channel inhibitors eg. amiloride Decrease permeability of luminal membrane to Na. As a result the driving force for K+ secretion is

eliminated, hence K+secretion ceases. Aldosterone antagonists eg. spironolactone

Aldosterone enhances the expression of the Na+, K+-ATPase and the Na+ channel involved in a Na+ K+ transport in the distal tubule . Spironolactone binds to this receptor blocking the actions of aldosterone on gene expression. Adverse effects: hyperkalaemia arrhythmias 1. Ca channel blockers (dihydropyridine amlodipine for BP, phenylalkylamine verapamil for heart) Block the voltage gated calcium channels in cardiac muscle and blood vessels decreases intracellular calcium reduction in muscle contraction SE: constipation, nausea, headche, rash, hypotension, edema, dizziness 2. ACEI (Captopril): blocks conversion of angI to angII, which is a potent vasoconstrictor and stimulates adrenal cortex to release aldosterone. SE: Hypotension, cough, hyperkalaemia, headache, dizziness, fatigue, nausea 3. AngII receptor blockers (Irbersartan): blocks the activation of AngII receptors vasodilation and reduced vasopressin SE: dizzinees, headache, hypotension

Pathology - Colon cancer and management principles (surgery, chemo, radio etc) - this one was a bit out of the blue Colorectal carcinoma

2nd most common, 2nd leading cause of death in Western country Colon cancer is more common in women and rectal is more common in men

Risk factors Age Smoking Dietary factors Low fibre content: stool bulk and transit time Increased intake of red meat: excretion of bile salts in the faeces promotes bacteria growth degrade bile salt and evidence shows that bile acids may act as carcinogens Associated to inflammatory bowel disease Especially ulcerative colitis Chronic extensive UC dysplasia of epithelium invasive cancer Familial adenomatous polyposis (FAP) Autosomal dominant Defect in APC gene on chromosome 5p21q Innumerous (500 ~2500) noeplastic polyps in the colon, primarily tubular adenomas Hereditary non-polyposis colorectal cancer (HNPCC) Autosomal dominant Previous gastric surgery

Investigations for CRC PR exam FOB FBC Iron deficiency anaemia Microcytic, hypochromic

Iron study: transferrin, ferritin, transferrin saturation Platelets increased in iron deficiency CRP/ESR Maybe Tumour markers CEA Mainly sued to track treatment than diagnostic Faecal occult blood EUCs - kidney LFTs liver function and coagulaopathy from liver disease could be a cause of bleeding Metastases causing obstruction can cause LFT abnormalities Without obstruction, there will be no change in LFT Coagulopathy secondary to liver disease can also cause bleeding Calcium/Phosphate bony metastatic can cause increase because they are lytic Staging CT chest, abdomen, pelvis Colonoscopy + biopsy Endoanal ultrasound or peolvic MRI for rectal cancer Barium enema old school not done.
Macroscopic Symptom Occult bleeding anaemia Fatigue Lethargy dyspnoea No change in bowel habit the food at this point is still in liquid form Change in bowel habit from obstruction Tenesmus Bright red fresh per rectal bleeding

Right side tumour

Ileocaecal junction Focal lesion in the caecum Polypoid or fungating along the wall exophytic mass obstruction not common

Left side tumour

Anorectal junction Annular shape central ulceration and raised edges Circumferential stricture

25% ascending (right), 60% descending (left)

Mode of spread haematogenously to liver by lympthatics into epicolic nodes pericolic nodes then to the nodes around the major vessels

Polyps Sessile vs pedunculated sessile is more worrying Villous vs tubular villous is worse I.e. Villous sessile adenoma is higher risk *** adenomatous polyposis coli (ADC) = familial adenomatous polyposis (FAP)

Mucosa is abnormally darked melanosus coli no significance associated with laxative use.

Colorectal adenocarcinoma All colorectal cancers are adenocarcinoma most arise from malignant transformation of benign adenomatous polyps. Tumour invading into the muscularis propria. Polyposis - Loss of cytoplasmic mucin,hyperchromasia, polarity changes, crowding, invasion, pleomorphism Irregularly shaped glands, increased mitoses, moderately differentiated (the glands are still a little tubular) Loss of crypts increased number of cells, At the top of the polyp - Pre-existing tubulo-villous adenoma. Lymphocytic infiltration. Desmoplasia (Host response Pink collagen laid down around the glands)

Feature of malignancy: Invasion, Irregular, glandular shape, increased N:C, Hyperchromatia, Pleomorphism

How would you stage this tumour, and what further information is required for accurate staging?

STAGING of CRC Tumour 0 = in situ not through the muscularis mucosa 1 = through the muscularis mucosa into the submucosa 2 = into the muscularis propria (externa) 3 = through muscularis propria but not into serosa 4 = through serosa and maybe other organs Nodes assessed on scanning and biopsy 0 = no nodes 1 = 1-3 nodes 2 = 4 or more Mets assessed on scanning CT and bone scan, x-ray etc O = no 1 = yes

Dukes staging T MN S g G up s ta e ro ing Stage I : T1 N0 M0; T2 N0 M0 A Confined to bond C ancer has begun to spre but is s in the inner lining. ad, till B Wall penetration, no nodes Stage II: T3 N0 M0; T4 N0 M0 C ancer has spread to adjacent organs but has not re ached lym node ph s. C Regional node involvement Stage III : any T, N1-2, M0 C ancer has spread to ly ph nodes, but has not bee carried to dis m n tant parts ofC body. node negative the Apical Stage IV: any T any N, M1 , 1 Apical node positive C ancer has bee carried through the ly ph sy n m stemto distant parts of the body.

C 2 D Distance metastatic

Management Surgery The tumour is removed along with adequate resection of the margins and pericolic lymph nodes. Continuity in restored by anastomosis when possible Carcinoma within a few cm of the anal verge may require abdomioperineal resection and formation of colostomy Radiotherapy Can be useful for inoperable or recurrent cancer by downstaging large bulky tumours Provides good palliation for troublesome symptoms of apin and rectal bleeding Chemotherapy Used in patients with metastatic or recurrent colorectal cancer Agent: 5-fluorouracil with folinic acid Immunoadjuvant Immunostimulatory agent levamisole in combination with 5-FU in high risk patients Locoregional chemotherapy Hepatic metastasis from colorectal cancer: 5-FU

Port Macquarie:

Anatomy (Vu and one of our locals) - Case: person comes in with headache or something - Pituitary macroadenoma compressing on optic chiasm (it was obvious, there was a brainstem model and papers with optic tracts lying on table) - Draw the nerve fibres from retina --> optic chiasm --> optic tract --> lateral geniculate nucleus --> occipital cortex

Identify on coronal T2 MRI - lateral ventricle, ICA, optic chiasm, sphenoid sinus, pituitary (a big macroadenoma)

- Identify on brainstem model - optic chiasm, lateral geniculate nuclueus (this one was retarded), optic tract, optic nerve

- Explain bitemporal hemianopia (compression of optic chiasm) vision is missing in the outer half of both visual fields. Information from the temporal visual field falls on the nasal (medial) retina. The nasal retina is responsible for carrying the information along the optic nerve, and crosses to the other side at the optic chiasm. When there is compression at optic chiasm the visual impulse from both nasal retina are affected, leading to inability to view the temporal, or peripheral, vision.

- Patient comes in a few months later with "ptosis, down and out gaze and fixed R pupil" - what has happened? (i.e. compress CN III in cavernous sinus etc)

- What nerve is palsied (CN III) - Explain the ptosis, down and out gaze and fixed R pupil A lesion of the oculomotor nerve will result in paralysis of the three rectus muscles and the inferior oblique muscle (causing the eye to rotate downward and slightly outward), paralysis of the levator palpebrae superious muscle (drooping of the eyelids), and paralysis of the sphincter pupillae and ciliary muscles (so that the iris will remain dilated and the lens will not accommodate) parasympathetic fibers run on the outer aspect of the nerve. - What else can give you CN III palsy (herniation syndromes) Causes: diabetes, giant cell arteritis, syphilis, posterior communicating artery aneurysm, uncal herniation - If you said herniation syndromes from raised ICP, then what causes ICP? Masses: haematoma, tumour, intracerebral haemorrhage or infarction causing oedema, abscess Diffuse brain swelling: encephalitis, hepatic encephalopathy, hypertensive encephalopathy, hypercarbia, severe head injury, status epilepticus CSF impairment: hydrocephalus, intraventricular tumours, meningitis, choroid plexus tumour

Microbiology: (Hazel mitchell and bill sewell) - One macro-investigation and one-microinvestigation from white book - just a blabbering fest really stupid and pointless and easy - Case history - flank pain, dysuria, febrile (39), septic shock (120 HR, BP was 90/60) - you had to notice the septic shock otherwise you failed - Diagnosis was septic shock secondary to pyelonephritis

- What organisms? E.Coli, Enterococcus faecalis, Klebsiella, Proteus mirabilis, Enterobacter - What investigations - MSU, blood cultures Urine dipstick leucocytes, nitrites MSU M/C/S FBC, EUC, CRP, blood cultures Ultrasound Cystoscopy/IVU - What is inside an anaerobe and aerobe tube? (I was like WTF? like culture for bacteria?!?!?!?) - How would you treat this gram negative sepsis Ciprofloxacin (quinolone) first line except in pregnancy Ampicillin and gentamicin - If you said gent (also amox i think), then they will ask you about gent monitoring and side-effects Gentamicin ototoxicity and nephrotocivity

Monitor gentamicin levels in plasma every 2 days

Pharmacology: (Margaret morris and a local) - Person comes in with thirst, pissing heaps, tired, overweight (BMI 32), fasting BSL of 8.6 - DD? (Diabetes mellitus)

- How to diagnose diabetes mellitus (OGTT etc) Fasting blood glucose >7.0 mmol/L 2 hours post glucose challenge (75g) >11.1 mmol/L Symptoms of diabetes + random blood glucose concentration >11.1 mmol/L - He was started on metformin - talk about how it reduces BSL, side-effects reduces hepatic glucose output and insulin resistance. No weight gain. absolute contraindication: liver or kidney dysfunction lactic acidosis SE: diarhhoea, cramps, N+V - How to monitor effectiveness of treatment (HbA1c, bsl, cholesterol etc etc etc) Urine tests glucose, ketones Blood glucose HbA1c - % of circulating haemoglobin with a glucose molecule attached over the life of the Hb (6 weeks) 24hr microalbuminaemia - If treatment wasn't working, would you add another drug (I said sulfonylureas) - talk about that drug Thiazolidinediones eg. Rosiglitazone: increases peripheral insulin sensitivity, reduces gluconeogenesis Sulphonylureas eg. Glibencamide: stimulates endogenous insulin secretion at the pancreas by inhibiting ATP sensitive potassium A glucosidase inhibitors eg. Acarbose: complex sugar that leads to reduced absorption of carbohydrate in the small intestine. - Asked if you knew any more drugs/insulin Short acting insulin Actrapid (6-8hrs) Intermediate insulin Protophane (12-24 hrs) Long acting Lantus (24-36 hrs) Pre mixed insulin Mixtard 30/70 (intermediate and quick acting insulin)

- Asked a bit about lifestyle dietary changes Low glycaemic index, high fiber, low carbs

Pathology: (Prof Kumar and a local)

- Lady comes in with haemoptysis, weight loss, tiredness, pleural effusion - DD? lung cancer - Further history and examination on lung cancer Hx: nature and severity of SOB, nature of cough, smoking history, industrial exposure to asbestos, overseas travel, family history Exam: accessory mm, clubbing, HPO, wheeze, lymphadenopathy, hepatomegaly, ascites - Investigations for lung cancer and findings FBC: anemia, leucocytosis EUC: increased Na with paraneoplastic syndrome CXR: consolidation, mass ABG: evidence of resp failure Sputum: M/C/S, acid fast bacilli, cytology - Got told there was a pleural drain done and maligannt cells seen - what further investigations to do? (Bronchoscopy and biopsy, CT etc) Bronchoscopy bronchial brushings, lavage, biopsy Ct abdomen liver and adrenal mets - How to explain the symptoms of lung cancer (haemoptysis, tiredness etc) Direct Spread: Invasion of pleura and ribs => chest pain and discomfort. Occasional pleuritic pain. Pleural effusion. Apical tumour => can involve the lower part of the brachial plexus (C8, T1, T2) causing severe shoulder pain and pain down the inner surface of the arm. Horners syndrome (ptosis, miosis, anhidrosis, enopthalmos) on affected side if sympathetic ganglion is affected. Hilar tumour => may involve recurrent laryngeal nerve, causing unilateral vocal cord paresis and a bovine cough Direct invasion of phrenic nerve => paralysis of ipsilateral hemidiaphragm Direct invasion of oesophagus => progressive dysphagia Direct invasion of mediastinum => pericardial effusion and malignant dysrhythmias Superior vena caval obstruction => early morning headache, facial congestion, upper limb oedema, jugular vein distension, distension of vessels on chest

Metastatic spread: Adrenal gland => often asymptomatic Bone => bone pain and pathological fractures Brain => personality changes, epilepsy, focal neurological deficits

Liver mets Spinal cord compression => back pain, paralysis, incontinence

Paraneoplastic syndromes: (esp small cell Ca) ectopic hormone secretion, various neuromuscular, haemoatological, dermatological symptoms eg. Ectopic ACTH syndrome, SIADH, hypercalcemia.

- Complications pneumonia, paraneoplastic syndrome, dissemination of mets, recurrent laryngeal n palsy, Horners syndrome - Causes of death pneumonia, pulmonary embolus, respiratory failure Resit questions I only know anatomy and microbiology

1) Microbiology around alcoholic cirrhosis (?????) and LFT - no microbiology was actually asked lol, they had to explain the LFT in the context of alcoholic cirrhosis, complications of cirrhosis and mechanisms Clinical Features

Portal Hypertension (disrupted architecture) Oesophageal varices, collateral vessels in abdominal wall, superior rectal haemorrhoids Ascites (also caused by Na retention and hypoalbuminaemia) Splenomegaly may get hypersplenism (rarely) Reduced Synthetic Function (cell death) Hypoalbuminaemia Ascites, oedema, drug toxicity Coag factors bleeding problems Reduced Metabolic Function (cell death and shunting) Drug toxicity Encephalopathy Oestrogen gynaecomastia, testicular atrophy, telangiectasia, enlarged nail lunulae, palmar erythema Aldosterone Na and water retention Reduced Excretion (cell death and dysfunction) Bile pigments Jaundice (late) Bile acids Steatorrhoea; Vitamin K deficiency (bleeding probs) Miscellaneous Parotid enlargement Skin lesions Complications (Decompensation) Bleeding Oesophageal Varices Portal Vein Thrombosis Slow flow due to hypertension thrombosis Presentation: sudden deterioration, ascites, bleeding varices Possible: bowel infarct (haemorrhagic), pyelophlebitis Infection (Shunting)

Septicaemia Spontaneous bacterial peritonitis Liver Infarction Bleeding leads to hypotension; presents with acute pain Hepatocellular Carcinoma Especially in HBV, HCV, Haemochromatosis Presentation: deterioration, pain, fever, tender/big liver Markers = AFP, CEA Renal Failure Hepatorenal syndrome altered systemic blood flow = reduced renal perfusion (no actual defect in kidney)

2) Anatomy - testicular torsion - I will follow this up The testicle is covered by the tunica vaginalis, a potential space that encompasses the anterior two thirds of the testicle and where fluid from a variety of sources may accumulate. The tunica vaginalis attaches to the posterolateral surface of the testicle and allows for little mobility of the testicle within the scrotum. In patients who have an inappropriately high attachment of the tunica vaginalis, the testicle can rotate freely on the spermatic cord within the tunica vaginalis (intravaginal testicular torsion). This congenital anomaly, called the bell clapper deformity, can result in the long axis of the testicle being oriented transversely rather than cephalocaudal. This congenital abnormality is present in approximately 12% of males, 40% of whom have the abnormality in the contralateral testicle as well.1 The bell clapper deformity allows the testicle to twist spontaneously on the spermatic cord. Torsion occurs as the testicle rotates between 90 to 1080 causing compromised blood flow to the testicle

>> Coffs: (we're not exactly sure of questions) >> Anatomy - Case history was about renal cell carcinoma, on CT identify - descending aorta, IVC, renal artery, renal vein >> Pharmacology - antihypertensives, mechanisms of actions, side-effects, contraindications

>> Microbiology/white book - community acquired pneumonia, how to take sputum and blood culture >> Pathology - Colon cancer and management principles (surgery, chemo, radio etc) - this one was a bit out of the blue >> >> >> Port Macquarie: >> >> Anatomy (Vu and one of our locals) >> - Case: person comes in with headache or something - Pituitary macroadenoma compressing on optic chiasm (it was obvious, there was a brainstem model and papers with optic tracts lying on table) >> - Draw the nerve fibres from retina --> optic chiasm --> optic tract --> lateral geniculate nucleus --> occipital cortex >> - Identify on coronal T2 MRI - lateral ventricle, ICA, optic chiasm, sphenoid sinus, pituitary (a big macroadenoma) >> - Identify on brainstem model - optic chiasm, lateral geniculate nuclueus (this one was retarded), optic tract, optic nerve >> - Explain bitemporal hemianopia (compression of optic chiasm) >> - Patient comes in a few months later with "ptosis, down and out gaze and fixed R pupil" - what has happened? (i.e. compress CN III in cavernous sinus etc) >> - What nerve is palsied (CN III) >> - Explain the ptosis, down and out gaze and fixed R pupil >> - What else can give you CN III palsy (herniation syndromes) >> - If you said herniation syndromes from raised ICP, then what causes ICP? >>

>> Microbiology: (Hazel mitchell and bill sewell) >> - One macro-investigation and one-microinvestigation from white book - just a blabbering fest really stupid and pointless and easy >> - Case history - flank pain, dysuria, febrile (39), septic shock (120 HR, BP was 90/60) - you had to notice the septic shock otherwise you failed >> - Diagnosis was septic shock secondary to pyelonephritis >> - What organisms? >> - What investigations - MSU, blood cultures >> - What is inside an anaerobe and aerobe tube? (I was like WTF? like culture for bacteria?!?!?!?) >> - How would you treat this gram negative sepsis >> - If you said gent (also amox i think), then they will ask you about gent monitoring and side-effects >>

>> Pharmacology: (Margaret morris and a local) >> - Person comes in with thirst, pissing heaps, tired, overweight (BMI 32), fasting BSL of 8.6 - DD? (Diabetes mellitus) >> - How to diagnose diabetes mellitus (OGTT etc) >> - He was started on metformin - talk about how it reduces BSL, side-effects >> - How to monitor effectiveness of treatment (HbA1c, bsl, cholesterol etc etc etc) >> - If treatment wasn't working, would you add another drug (I said sulfonylureas) - talk about that drug >> - Asked if you knew any more drugs/insulin >> - Asked a bit about lifestyle dietary changes >> >> Pathology: (Prof Kumar and a local) >> >> - Lady comes in with haemoptysis, weight loss, tiredness, pleural effusion - DD? lung cancer >> - Further history and examination on lung cancer >> - Investigations for lung cancer and findings >> - Got told there was a pleural drain done and maligannt cells seen - what further investigations to do? (Bronchoscopy and biopsy, CT etc) >> - How to explain the symptoms of lung cancer (haemoptysis, tiredness etc) >> - Complications >> - Causes of death >> >> Resit questions I only know anatomy and microbiology >> >> 1) Microbiology around alcoholic cirrhosis (?????) and LFT - no microbiology was actually asked lol, they had to explain the LFT in the context of alcoholic cirrhosis, complications of cirrhosis and mechanisms

Diagnostic Tests Exam Questions For this station, the examiners will look at your white book and ask you questions about one of your anatomy cut-ups and one of the laboratory investigations that you have seen throughout the year. Most examiners will choose which ones to ask you questions about. Basically they will ask you the ins and outs of the anatomy cut-up and laboratory investigations that you have seen i.e. purpose of test, processes involved, results obtained, interpretation of results, details about patient etc. The second part of the station will be a set case history. Examples are listed below: 1. Renal failure/Renal physiology, 47yo person with peripheral leg oedema, hypertension, tachypnoea, urinalysis with red cells, protein and casts. Also get shown EUC results.

What features are consistent with acute renal failure? fluid overload: sudden increase in urea and creatinine. Maybe nephritic syndrome Tachypnoea fr metabolic acidosis.

How do you calculate GFR? 24 hr creatinine clearance GFR = (urine conc x vol)/ plasma conc

Or Cockcroft-Gault formula

How does hyperkalaemia occur? H+ and K+ vie for secretion at DCT So secretion of K+ decrease during acidosis So what causes acidosis? Metabolic acidosis in as unable to secrete H+ . so try to reabsorb as much bicarb

RTA Definition: RTA is an impaired secretion of hydrogen ions / resorption of bicarbonate ions . This causes metabolic non-anion gap (hyperchloremic) acidosis

METABOLIC ACIDOSIS Most severe type: ClassicType 1 RTA distal Hypokalaemic RTA defective H+ secretion ( due to K+/H+ exchanger at alpha intercalated cell of CD) autoimmune Failure to excrete in respi/renal failure will cause hypercalciuria, renal stones & nephrocalcinosis. Hypokalemia (K+ and H+ compete for secretion) Type 2 RTA proximal Hypokalaemic RTA defective HCO3 regeneration ( K+ & H+ vies for secretion at proximal part) Fanconi Syndrome renal loss of HCO3- , increase retention of H+, Increase secretion of K+. Type 4 RTA distal Hyperkalaemic RTA defective H+ secretion. Manifested as Hyperkalaemia. low aldosterone and kidney resistant to it so no K+ secretion. results from aldosterone deficiency or unresponsiveness of the distal tubule to aldosterone. Because aldosterone triggers Na resorption in exchange for K and H, there is reduced K excretion, causing hyperkalemia, reduced ammonia production, and reduced acid excretion. Type 4 is also called hyperkalemic RTA and is caused by a generalized transport abnormality of the distal tubule. The transport of electrolytes such as sodium, chloride, and potassium that normally occurs in the distal tubule is impaired. This form is distinguished from classical distal RTA and proximal RTA because it results in high levels of potassium in the blood instead of low levels. Either low potassiumhypokalemiaor high potassiumhyperkalemiacan be a problem because potassium is important in regulating heart rate. Type 4 RTA occurs when blood levels of the hormone aldosterone are low or when the kidneys do not respond to it. Aldosterone directs the kidneys to regulate the levels of sodium, potassium, and chloride in the blood. Type 4 RTA also occurs when the tubule transport of electrolytes such as sodium, chloride, and potassium is impaired due to an inherited disorder or the use of certain drugs.

What complications are we worried about with high potassium? Arrthymia, cardiac arrest

Tall t wave. Asytole. Tx: Give calcium gluconate, iv insuline glucose, calcium resonium to bind K+ in gut, hemodialysis How does potassium do that (i.e. make an arrhythmia)?

1. Elderly divorced lady who drinks 6-8 glasses/day and smokes 10-15 cigarettes per day with very poor liver function What is your provisional diagnosis?

Alcoholic liver-hepatitis, fatty changes, cirrhosis, HCC Can you interpret the laboratory findings given here? If AST> ALT liver injury by chronic alcoholism If GGT raised and no change in ALP: due to small alcohol intake. Why do you think her transaminases are not elevated? No acute injury Why do you think her globulin may be raised? Immunoglobulins (if not available, raised immunoglobulins may be suggested by a raised globulin fraction (total protein minus albumin))as blood (bact Ag) shunt from reticuloendothelial system liver, to lymphoid tissue How does alcohol cause a raised GGT? GGT raises when biliary stasis. Together with ALP. But if sold GGT raise, Alcohol cirrhosis cause obliteration of bile flow as a reulst What are other tests you can do to determine the livers synthetic function? Bleeding time. If she was to bruise, what would be some of the reasons behind this? Vitamins A, D, E, K deficiencies as bile salt aid in fat absorption which is req for absorption of the vit amin Vit k derived clotting fact 2, 7, 9, 10 As cholestasis and bile acid retention. Bile acid needed for Vit K absorption How does alcohol and cirrhosis cause portal hypertension and ankle oedema?

Ankle edema is due to portal hypertension. Cirrhosis = irreversible destruction of the liver architecture by fibrosis and regeneration of nodules of hepatocytes As cirrhosis causes obliteration of portal flow due to the scarring and fibrosis as a result the hypertension and portal system are valveless so there is a retrograde flow into the system. Also reduce synthetic function of albumin so hypoalbuminemia so ascitis and edema What would be an ultimate cause of death?hepatic encephalopathy confusion, stupor, coma What do you think her electrolytes might look like? I.e. sodium levels? Why? As accumulation of nitrogenous waste splannic venous dilation pooling of bld decrease perfusion in other organs renal artery react by vasocon decrease glomerular flow and RAAS activated retention of water and dilutional hyponatremia due to inability to excrete free water resulting from high levels of ADH and aldosterone. Why does she have a tense abdomen? Why the shifting dullness? Ascitis from portal hypertension and hypoalbuminemia Why does splenomegaly occur? congestion What are other complications of portal hypertension? Ascitis, hemorrhoids, caput medusa, esophageal varices encephalopathy How would the patient probably have presented? Haematemesis

Ascites (fluid retention in the abdominal cavity) is the most common complication of cirrhosis and is associated with a poor quality of life, increased risk of infection, and a poor long-term outcome. Other potentially life-threatening complications are hepatic encephalopathy (confusion and coma) and bleeding from esophageal varices Others: hepatorenal syndrome = ARF

Other tests to order (INR) prothrombin time, serum albumin/globulin

1. UTI, 28yo sexually active female presents with burning, freq of urination and discharge. Ix: MSU MCS, bld culture Culture with chromagar How would you do a MSU? Why is midstream important? What organisms can it be contaminated with? Normal skin flora. Staph aureus, epididermitis. Midstream or 'clean catch' urine, catheter or bladder aspirate specimen in a sterile container sent to the laboratory immediately, or refrigerated for up to 24 hour What can we do with urine after collection? Freeze it and send for culture immediately as its a quantitative test. What can you find on a MSU? Microorganism, red cell, white cell, epithelial cell, cast What is a positive result for UTI? 10^ 4 organism of GN rod /ml

The MSU showed 10^8 gram +ve cocci, 10WBChpf, <10RBC and no epithelial cells. What does this mean? This indicates infection.

Common UTI organisms.Ecoli, proteus mirbilis Other GN coagulase ve staphylococci GP+ve: candida, klebsiella, in sexually active F staphylococci How would you treat? Trimethoprim 200mg/12 h po (3d) or cephalexin 1 g/12 h Drink plenty of water, urinate often If you left the urine out for 4 hrs in room temp, what would happen? What organisms can grow? How would this affect the results?the organism will multiply by itself. Cuz the test result rely on the quantity so the result will be false positive What about if the patient has been in hospital for 2 weeks and is catherised what organisms?E coli How do we test for sensitivity? With antibiotic susceptibility test generally > or equal to 6 mm zone size to be significant For vacomycin > or equal 2mm to be significant

What antibiotics would you test resistance for in this case (gram positive cocci)? Penicillin, augmentin, ampicillin, cephalosporin, trimethoprim, vancomycin, methicilin, gentamicin With broad spec gentamicine including pseudomonas aeruginosa and staph But if wanna cover strep and anerobe (enterococci), use amox Genta + amox MRSA

http://web.med.unsw.edu.au/cdstest The nine steps followed in performing the CDS Test are represented diagrammatically in Figure 2.1. Further details of particular aspects of the method, including preparations necessary before the performance of the actual test, are set out below:

1. Stab a fresh colony with a straight wire.

2. Rotate the straight wire in 2.5ml saline.

3. Inoculate the pre dried plate.

4. Distribute inoculum by rocking.

5.

Remove excess inoculum.

6. Dry uncovered at room temperature (max. 30 min.).

7. Load plate with antibiotic discs.

8.

Incubate for 18 hours.

9.

Measure annular radii.

1. SLE case in Path CD under multisystem disease, 34 year old women with SLE with previous miscarriages. Presented with new rash on face, lethargy, malaise and arthralgia. What are your differentials? PD: flaring of SLE DD: RA,, scleroderma hepatitis? , What are some of the connective tissue diseases you know of? RA, scleroderma, sjogrens syndrome, raynauds Explain significance of antinuclear antibody in this case?

Its not specific but sensitive. Detects disease activity? What was significance of the women having previous miscarriages? (as SLE is a secondary antippl syndrome) ntiphospholipid syndrome which specific antibodies attack substances in the blood, causes tiny blood clots to block the blood supply to the placenta. Others believe that having antiphospholipid syndrome may interfere with the fertilized eggs ability to implant in the lining of the uterus. What is the pattern on this immunofluorescence slide (from Path CD) Diffuse pattern anti DNAand histone = drug induce lupus Anti DsDNA: rim = SLE Anti ribonucleoptn: speckled = sjogren/scleroderma Anti RNA: nucleolar

Basic: FBC, UEC, ESR, U/A: Microscopic examination of urinary sediment Immunology: ANA (+ve in ~ 100%) and antibodies to extractable nuclear antigens (standard serological test)

specific autoAb : Anti-double stranded DNA antibodies, anti Smith (anti-Sm), antiribonucleoprotein (anti-RNP) antibodies aka anti Ro(Ab to Ro, La and U1 ribonuclear protein help define overlap syndromes (eg with Sjogrens)) Antiphospholipid (anti lupus coagulant, and anti cardiolipin) Anti PPL +ve (as SLE is a secondary antippl syndrome) abnorm serum lvl of IgG or Ig M anticardiolipin Ab have false +ve syphilis serology fr Ig G anticardiolipin Ab +ve for lupus anticoagulant using standard mtd False +ve serological test for syphilis +ve comfirmed by T. pallidum immobilization or fluorescent treponemal Ab absorption test

complement levels ( C3 & C4 C3d due to degradation pdt of C3) rheumatoid factor (+ve in 30-50%) Imaging: Chest x-ray, ECG, echocardiography CT scan brain infarcts/hemoorhage/cerebral atrophy Skin bx

The LA and aCL (particularly high titre IgG) are associated with clotting, foetal loss, thrombocytopaenia and valvular heart disease High serum levels of ANAs and anti-dsDNA and low levels of complement usually reflect disease activity, especially in patients with nephritis

1. Infective endocarditis, 35yo woman presented to ED unwell with fever following a recent dental procedure. Ex: unwell and splinter haemorrhages in nail beds. Present your differential diagnosis inc: IE, sepsis, bleeding disorder What investigations are appropriate? (Imp: FBC, Blood cultures, CXR, Echo (vegetation), ECG (silent infarct/conduction defet) Discuss how a blood culture is taken: (Emphasis here was on sterile technique) 3 samples over 3-6 hrs from diff site Have a look at this result from the blood culture, discuss: Result: Bottle 1a/1b positive for Streptococcus epidermitis (which is an alphahemolytic oral commensal), bottle 2a positive for staph epidermitis, bottle 2b positive for strep epidermitis. So IE fr dental work

6. Picture of bruising, menorrhagia of young woman and mild splenomegally DD What are the diff types of bleeding disorder

What investigation will u order Blood film shows NN anemia, thrombopenia, blast and myelocyte, increase WBC What further investigation will u order Pneumonia with staph aureus Abscess formation what causes it hemolysin and coagulase

New Questions 2010


1) Lymphoma (very similar to case 23). History of sore throat and palpable lymph nodes. I think there may have been some splenomegaly. a. What is your ddx? b. What further qs would you ask? c. What investigations would you order? d. What would you see on the lymph node biopsy? e. What is the staging system used? Ann-Arbour 2) Acute Myeloid Leukaemia (very similar to case 29) a. What is your Ddx? b. What investigations would you like to order? c. They give you a FBC, with diff and film. What are the abnormalities? i. Why is it AML? As there are blasts, all the myeloid lines are affected. Lymphocytes are not affected. As bone marrow is replaced with myeloid leukaemic cells, there is a drop in RBCs and platelets. ii.What are platelets derived from? Megakaryocytes d. What other investigations would you like to do? i. Bone marrow aspirate and trephine. Will show hyperceullarity. Increased myeloid to erythrocyte ratio. Will contain abnormal myeloid precursor cells. Can be tested with immunophenotyping, for gene defect.

Pain management The station started by askin you to look at the med chart and comment on that For that patient it was IV morphine 8 hrly may not be enuf so give 4 hrly as half life of morphine is 2-4 hrs (thats y the patient complain about pain) If its IM change it to IV faster effect IM oso more painful So can give Sc 4 hourly Well I was commenting about the gastric immotility that morphine can caused which is not so good for this post surgical patient (ileus). But I dont think thats the answer they wanna hear but if you want you can mention too.

If pain still unmanageable change it to fentanyl or tramadol? or add on NSAIDS

May have serotonin syndrome Other options including NSAIDS. Not given in this patient but in the campus day did mention bout givin it but you can mention need to take note cuz this patient has Cardiac failure may precipitate renal failure or the cardiac failure. COX 1 for GI protection platelet aggregation, renal function bld flow COX 2 PG for inflammation (fever, pain, headadace), carcinogenesis So get a COX 2 selective inhibitor. Eg celecoxib Becareful of patient with heart failure may further decrease renal blood flow with use COX 1 Check renal function Major side effects: GI symptoms : N+ V Dyspepsia, GIT bleeding (Esp in elderly) Hypersensitivity (rash, urticaria and angioedema, photosensitivity, asthma (Esp aspirin) Renal dysfunction as reduced PGE2 cuases lack of vasodilation to compensate the vasoconstriction of adrenaline and angiotensin hence renal ischemia Hypertension: MOA same as renal During pregnancy, inhibition of PGE2 and PGF 2alpha causes prolongation of gestation May cause closure of the ductus arteriosus and impaired fetal circulation in utero.

Opiods SE (they ask bout the side effects)

They oso ask about wat should you consider if you give opioid:

Then they say if a drug dependent person came in how would u manage. I guess the answer is slowly increase the dose and as they need increase dosage due to dependence watch out for toxicity. and mayb can use methadone and even continue using it after as maintenance programe.

Physical dependence -Increase dose needed -Difficult to stop -Spend lotsa time thinking, looking ,using and getting over the effects of using it Withdrawal, -Start after a few hrs -Last several days, nt life threatening -Vomit, diarrhea, runny nose, perspiration Opiates (morphine, codein, heroine) Short term effect: +ve pain relief & high, pit pt pupils -ve nauea, vomiting, sleeping, physical dependence Long term effect: loss of sex drive, infertility in F, irregular period infections/ ulcer, malnutrition, vein damage.

So what you can do? In ED

In long run

They oso ask me to comment about the side effect of paracetamol and ondansetron which is in the chart. Paracetamol PO: may not be suitable for him cuz he juz had a bowel surgery. May need to be NBM Safe analgesic and antipyretic 2-4 g/day for adults) But potent hepatoxin in large overdose Factors affect toxicity: cytochrome P450 inducer: alcohol, isoniazid rifampin (TB) Phenytoin, Phenobarbital, carbamazepine Depletion of gluthatione and glucuronic acid storage, fasting, choric alcoholism When taken at the recommended dose, side-effects of paracetamol are rare. Skin rashes, blood disorders and a swollen pancreas have occasionally happened in people taking the drug on a regular basis for a long time. One advantage of paracetamol over aspirin and similar drugs (eg ibuprofen and diclofenac) is that it won't upset your stomach or cause it to bleed. A paracetamol overdose is particularly dangerous because the liver damage may not be obvious for four to six days after the drug has been taken. Even if someone who has taken a paracetamol overdose seems fine and doesn't have any symptoms, it's essential that they are taken to hospital urgently. An overdose of paracetamol can be fatal. Common SE: rare Uncommon SE: indigestion, nausea, rash, hives, allergy, drug fever Interactions with other medicines

You may need to adjust your usual dose of anticoagulants (eg warfarin) if you take paracetamol regularly. Check with your anticoagulation clinic. Otherwise there are no serious interactions between paracetamol and other drugs. Odansetron Use sc instead of IMI as IM is painful and absorption is erratic so not recommended ( I think in the paper its IM) Works by anti the effect of serotonin which causes vomiting SIDE EFFECTS: Diarrhea or constipation, headache, lightheadedness or drowsiness (uncommon), as well as blurred vision may occur. If these effects persist or worsen, notify your doctor promptly. Very unlikely but report promptly: chest pain. In the unlikely event you have an allergic reaction to this drug, seek immediate medical attention. Symptoms of an allergic reaction include: rash, itching, swelling, severe dizziness, trouble breathing. If you notice other effects not listed above, contact your doctor or pharmacist.

Etpg Post op pain mx

This should include regular paracetamol, and often some form of opioid. The addition of an NSAID, other adjuvants, or regional techniques using local anaesthetics, may be appropriate.

After minor surgery (including some day-surgery procedures), effective analgesia can usually be obtained without using opioids, thus avoiding their adverse effects. After major surgery, opioid analgesics are usually required.

Pathology Exam Questions 1. Lung Cancer, 63yo male presents with two week history haemoptysis and a 2 month history of weight loss. He has smoked 2 packets of cigarettes a day for 45 years. Provisional and DDx, does pneumonia usually present with haemoptysis (have to say lobar pneumonia, and has to form abscess/invade blood vessel eg. staph) Pd: lung cancer or mets Dd: TB, bronchiectasis, emphysema,necrotising pneumonia that form abscess and invade blood vessels Bronchopulmo/heart/hemato Broncho pulmo: inflammatory/neoplasms, infarct/trauma 1. Investigations to rule out other causes and confirm your diagnosis?

FBC: leukocytosis UEC Sputum ABG LFT CXR, bronchoscopy/bx CT 2. What do you look for in sputum? Malignant cells, microorganisms 3. What are the three types of sampling specimens that can be taken during bronchoscopy and how is the procedure done? Complications of procedure. Biopsy, bronchoalveolar lavage, endobronchial brushing Bronchoscopy is a technique of visualizing the inside of the airways for diagnostic and therapeutic purposes. An instrument (bronchoscope) is inserted into the airways, usually through the nose or mouth, or occasionally through a tracheostomy. This allows the practitioner to examine the patient's airways for abnormalities such as foreign bodies, bleeding, tumors, or inflammation. Specimens may be taken from inside the lungs: biopsies, fluid (bronchoalveolar lavage), or endobronchial brushing. Bronchoalveolar lavage (BAL) is a medical procedure in which a bronchoscope is passed through the mouth or nose into the lungs and fluid is squirted into a small part of the lung and then recollected for examination. Besides the risks associated with the drug used, there are also specific risks of the procedure. Although the rigid bronchoscope can scratch or tear airway or damage the vocal cords, the risk of bronchoscopy is limited. Complications from fiberoptic bronchoscopy remain extremely low. Common complications include excessive bleeding following biopsy. A lung biopsy also may cause leakage of air called pneumothorax. Pneumothorax occurs in less than 1% of cases requiring lung biopsy. Laryngospasm is a rare complication but may sometimes require intubation. Patients with tumors or significant bleeding may experience increased difficulty breathing after a bronchoscopic procedure, sometimes due to swelling of the mucous membranes of the airways. 4. What types of lung cancer are there? NSCL, SCL Grading and staging. Investigations for staging. Where in the lung do they arise (bronchi vs pleura and parenchyma vs epithelium)? What symptoms are produced? Bronchial type: sq30, small cell25 (assoc with smoking) And lobar type: adeno30, large cell15 Alveolar < 1 % 5. Treatments Lungresection, Chemo,radiotherapy Smoking cessation 6. Discuss pot of lung cancer. Bronchial ca squamous cell type.

7. How does lung cancer spread and what organs can it affect? Consequences of invasion. Hematogenous, lymph, local invastion, Local invasion: obstruction: atelectasis Local invasion into tissue: bleeding, to recurrent laryngeal nerve Decrease cardiac return: SVC obstruction 8. Where does it spread locally? Local invasive to occlude bronchus --> collapse Mediastium --> SVC obstruction (how would this present clinically?) increase jvp, engorged face Recurrent laryngeal nerve --> hoarseness What about an apical tumour?? --> brachial plexus (erbs palsy) What is horners syndrome? Invasion of cervical sympathetic plexus: anhydrosis, miosis(constricting pupil), drooping eyelid, enopthalmus, upside down ptosis (upside down droopin of eyelid)

9. Local effect: 10. Central tumor obstruction atelectasis & post obstruction pneumonia, emphysema 11. Peripheral tumour no abnorm on PE 12. Apical tumour aka pancoast tumor, presses on cervical sympathetic plexus 13. Horners syndrome: pupil constriction, drooping eyelids (ptosis), loss of sweating of unaffected size 14. If press on brachial plexus Severe pain along ulnar nv muscle wasting in hands hypothenar & thenar 15. 16.Local effect of invasion: 17. Phrenic nerve invasion diaphragm paralysis 18.Cervical sympathetic ganglion -> horner syndrome 19. Recurrent laryngeal nerve hoarseness 20.Invasion: 21. Pleura pleural effusion due to lymphatic blk 22. Chest wall rib destruction 23. Esophageal dysphagia 24. SVC SVC syndrome (venous congestion) 25. Airway obstruction pneumonia, abscess, lobar collapse 26. 27.Paraneoplastic syndrome: 28.SIADH: syndrome of inappropriate ADH 29.Cushing syndrome: ACTH 30.Hypercalcemia: parathyroid 31.Hypocalcemia: calcitonin 32.G ynaecomastia: gonadotrophins 33.Carcinoid tumor: serotonin 34.CVS: thrombophlebitis, marrantic endocarditis 35. 36.Local invasion, lymphatics, hematogenous

37.What conditions would affect his prognosis? Mets , grading, type of tumor 38.What factors would affect decision to operate?

39. Stage I: Node 0: so either asymptomatic or local effects, invasion (nerve), local obstruction 40.Stage II:

41.N1peribronchial/ipsilateral node hilum 42.T3 involves the chest wall, diaphragm, mediastinal pleural, pericardium or < 2 cm fr but not at carina 43.Chest wall erosion, phrenic nerve palsy, pleural effusion, 44.Stage III 45. N3 contralateral mediastinum or hilum, scalene or supraclavicular SVC syndrome, lymphadenopathy 46.T4: mediastinum, heart great vessels (SVC), trachea (dyspnoea), oesophgus (dysphagia), vertebral body, carina or malignant effustion 47. Stage IV skin mets (acanthosis nigricans), brain, bone (bone pain, anemia, +Ca), liver (hepatomegaly) 48. 49.In summary, 50.Stage I mostly asymp or local effects 51.Stage 2/3: obstruction, invasion 52.Stage 4: mets 53.

Mets
The overall prognosis for lung cancer is poor. The median survival time for limited-stage SCLC is 20 mo, with a 5-yr survival rate of 20%. Patients with extensive-stage SCLC do especially poorly, with a 5-yr survival rate of < 1%. Recently, patient survival has improved for patients with both early and later stage NSCLC. Evidence shows improved survival in early-stage disease when platinum-based chemotherapy regimens are used after surgical resection

54.Interpret spirometry result. Normal values, type of pattern. How would CAL/COPD affect his chances of surgery? Asthma/COPDdecreaseFEV1/FVC Lung fibrosis: same ratio as both decrease Poor respiratory function will affect prognosis of surgery. Like intubation and stuff Value according to age and gender and ethnicity. Normal around 0.8 1. 19 yo female medical student with chronic bloody diarrhoea with previous episode one year ago. What is your provisional diagnosis and what are your differential diagnoses?
Ulcerative colitis/Chrons Infective gatroenterities: eg.shigella, hpe a, nsaids, hemorrhoids Ix: ESR

UC/CD

What further questions would you like to ask her and what would you like to look for on examination? Is there mucus How long is the episode Previous episode: any treatment, respond

Symptoms of anemia Stomachpain,extraintesstinal manifestation of ibd uc/Cd family hx

PE: Exam: abdo, malnutrition, stomatits, glossitis

Extra GI manifestation of UC/CD scleritis, polyarthritis

If she was known to have taken antibiotics recently, how might that affect your diagnosis? Infectious gastroenteritis due to the wipe out of normal flora by ab
the pathophysiology of Clostridium difficile-induced "pseudomembranous colitis Broad spectrum Antibiotic-associated colitis characterised by the formation of an adherent inflammatory exudate (pseudomembrane) overlying sites of mucosal injury Organism: Gram positive bacillus, anaerobe Pathogenesis: Normal commensal of the gut, overgrows other intestinal flora in antibiotic-treated patients due to reduction in normal protective flora Produces two exotoxins: Toxin A = enterotoxin also potent granulocyte chemoattractant: causes intestinal secretion and acute inflammation Toxin B = cytotoxin (damage GI epithelial cell) binds to receptors on epithelial cells, inactivate the family of RhO cytoplasmic proteins, thereby causing disaggregation of actin microfilaments and cell retraction Morphology: Macroscopic: plaque-like adhesion of fibrinopurulent-necrotic debris and mucous to damaged mucosa (not a true membrane: the coagulum is not an epithelial layer) Microscopic: Surface epithelium is denuded Superficial lamina propria = lots of PMNs, occasional capillary fibrin thrombi Damaged crypts = distended with mucopurulent exudate erupts out of the crypt mushrooming cloud; adheres to damaged surface clouds coalesce to form pseudomembrane over the ulcer Clinically Acute or chronic diarrhoeal illness C. difficle toxin B detected in stool Treated well but 25% relapse

Investigation Stool specimen Toxin detection ( EIA) toxin B

Peripheral stigmata of IBD, why look for it, what information would it give you?

Pathology pot of UC: discuss features, rectum doesnt look involved how is this significant?
Starts in rectum spreads up continuos (pancolitis- entire colon severe) -Backwash ileitis in severe pan colitis terminal ileum dvp inflammation due to incompetence of ileocecal valve. -Crypt abscess- common but nt specific -pseudopolyps -No thickening, narrowing, granuloma

Disease progess What other investigations would you do now? What would they show you?

Colonoscopy. stool culture to confirm if its not gastroenteritis. If the disease progresses what other complications can take place in the bowel? Toxic megacolon, peritonitis, epithelial dysplasia canncer What extra-intestinal manifestations are there?
Extraintestinal manifestation( same for UC) Migratory polyarthritis, sacroilitis, ankylosing spondylitis, erythema nodosum, clubbing. Uveitis(eye), hepatic cholangiitis, renal disorders ( entrapping of ureter by inflammation)

Oral ulcer, 1. Breast cancer, 42 year old woman who discovered a breast lump 15mm in diameter, cousin had breast cancer. Pdx: Breast cancer Ddx Benign: fibrocystic changes(blue dome), proliferative breast disease Benign neoplasm: fibroadenoma(breast cyst), lipoma, phyllodes tumour Malignant: LCIS, DCIS, invasive LC and DC Further history and exam would

Character, onset, progress of lump; d/c fr nipple bloody , foul smelling Any other systemic effects Does it change with menstrual cycle Previous cancer/radiation exposure Significance of family history and breast cancer.

Autosomal dominant gene Mutate PRCA1, PRCA2, p53 10% hereditary assoc with BRCA 1 or 2 80-90% if both genes Risk factors family history there is an increased risk, however, most breast cancers sporadic:

5-10% of breast cancers are related to inherited mutations, eg p53. First-degree relative with breast cancer 1.2-3.0x risk Premenopausal 3.1; if premenopausal and bilateral 8.5-9.0x Postmenopausal 1.5x; if bilateral 4.0-5.4x BRCA-1 and BRCA-2 accounts for a small percent of familial breast ca; Menstrual history age at menarche <12 1.3x . age of menopause >55 years Pregnancy risk increases with increasing age, Benign breast disease Proliferative disease 1.9x Proliferative disease with atypical hyperplasia 4.4x Lobular carcinoma in situ 6.9-12.0x
Genetic factors account for breast cancer is less than 10% of cases. Most r sporadic. So attempts at its prevention are somewhat ineffective. Chemoprevention with tamox for hi risk is currently on trial. But concersn bout risk of endo can on long term use Prophylactic mastectomy with or w/o breast recon is occasionally performed for v. hi risk indiv who demand it Come for regular screening.

PE: general appearance(peud orange, retraction), the lump, nipple, axillary/infraclavicular


Other risks -Sex F 100X > M -Increasing age. mean age at dx is 60. Uncommon in < 30 -Past hx of breat cancer. May be par t of a multifocal origin of the first cancer or entirely new cancer -First deg relative who have breast cancer esp if under 50 -Prev hx of benign proliferative disease with cellular atyple, multiple appillomatosis, atypical ductal and lobular hyperplasia, lobular CIS -Other: nulliparity at 40, prev breast irradiation, younger age of menarche. Info on HRT conflicting but short duration < 5 yrs appears safe. -OCP doesnt seem to be assoc with any increased risk. -Genetic BRCA1/2, tp3, Ashkenazi jews -Hormonal: endogenous (tumor, parity, obesity, reproductive life), exogenous, hormonal imbalance, -Geographical/environmental/iatrogenic exposure Mutation in gen BRCA 1 and 2(tumor suprressor). Will have ovarian cancer as well. BRCA 1: lifetime risk of breast cancer 70-80%, ovarian cancer 30-40% BRCA2 less likely to cause ovarian cancer. May cause male breast cancer in some family Screening: mammogram, ovarian ultrasound with analysis of CA 123 and CEA in those at moderate risk. Debate about cost-benefits of screening and risks of radiation exposure fr regular mammography. Hi risk: prophylactic mastectomy and oopherectomy.

What factors determine prognosis?


Tumour size The histological type of the tissue

Local lymphovascular invation The hormone receptor sensitivity (HER 2 overexpression) Axillary lymph node status sentinel

HER2-positive breast cancer is a breast cancer that tests positive for a protein called human epidermal growth factor receptor-2 (HER2), which promotes the growth of cancer cells. In about one of every three breast cancers, the cancer cells make an excess of HER2 due to a gene mutation. This gene mutation can occur in many types of cancer not only breast cancer. HER2-positive breast cancers tend to be more aggressive than other types of breast cancer. They're also less responsive to hormone treatment. However, new treatments that specifically target HER2 are proving to be very effective:
Discuss features of breast cancer pot. Color, size, well circumscribed? In the lobe or duct or invasive.

1. Diabetes, Case Protocol 31 How would you interpret the history and physical findings? Increasing weakness and lethargy Thirst Vague abdo pain Pale, thin, Drowsy Dry mucosa Ketotic fetor
History Vital signs Tachycardia, hypotension hypovolaemia Tachypnoea due to acidosis stimulation of chemoreceptors (metabolic acidossi Low grade temperature possibly due to infection

Explain the vital signs - hypovolemic shock,


Weakness and lethargy from wasting and hyperosmolality (loss of calories) Thirst blood glucose levels too high for renal resorption glycosuria osmotic diuresis fluid and electrolyte loss Abdominal pain gastric stasis and distention Vomiting often occurs in diabetic ketoacidosis. Thin accelerated breakdown of fat and muscle due to deficiency of insulin- loss of calories Drowsy due to hyperosmolality, hypovolemia and dcrease perfusion to brain Dry mucosae, pale - dehydrated Ketotic foetor smell of ketones

is BSL diagnostic of DMKA? Diagnosis of diabetic ketoacidosis Hyperglycaemia (dipstick) + ketonaemia In symptomatic patients, a single elevated blood glucose 11.1 mmol L-1, measured by a reliable method, indicates diabetes. Heavy ketonuria (dipstick) + acidosis

Any of the following are grounds for the diagnosis of DM: Fasting plasma glucose: 7.0 mmol/L or 126 mg/dL (upper limit of normal = 110mg/dL). 2 hours post glucose challenge plasma glucose 11.1 mmol/L or 200 mg/dL. Symptoms of diabetes + random blood glucose concentration 11.1 mmol/L or 200 mg/dL.

Glucose tolerance test Normal < 5.6 IFG FPG btw 5.6-6.9 IGT 2hPG 7.9-11 Diabetes FBG > 7 2hPG >11.1 Random >11.1

Elevated potassium acidosis, insulinopenia and elevated glucose cause a shift of K out of cells; therefore raised levels despite depleted K stores due to diuresis;

Hyperkalaemia - why?

What investigations would you perform? ABG UEC FBC Dipstick Interpret ABG results (metabolic acidosis with partial respiratory compensation)

pH low, base excess negative metabolic acidosis. This is consistent with low bicarbonate. PaCO2 low hyperventilation to blow off carbon dioxide compensatory respiratory alkalosis

(CLINICAL CHEMISTRY Na decreased, Cl decreased, K increased, Urea Increased, Creatinine Increased, Osmolality increased) Are the glucose and osmolality results diagnostic of DM or do you need to perform more tests? Why is the potassium level so high? What type of renal failure is this? Prerenal ARF Asked about the pathophysiology of Type 1 Diabetes and DKA.

The exact aetiology of insulin dependent diabetes is unknown, but we know: 3 mechanism T cell and autoantibodies Infection Genetic susceptibility HLA DR3 DR4 There is a T cell mediated autoimmune destruction of B cells of the pancreatic islets Islet cell antibodies appear in the circulation in the first few years of life and many years before diagnosis. Whilst type 1 diabetes is not genetically predetermined, but increased susceptibility to the disease may be inherited. Associated with HLA-DR2 or DR4 in >90% There may be an environmental cause, possibly a viral infection, however, no aetiological agent has been proven. Pt are vulnerable to hypoglycaemic episodes (due to tmt) / ketoacidosis (although almost exclusively occur in IDDM and is stimulated by severe insulin deficiency coupled with absolute or relative increases of glucagon) (Insulin deficiency excessive breakdown of adipose stores increased FA within liver oxidises (acetyl CoA) ketone bodies (rate of production>rate of excretion) ketonemia + ketonuria dehydration + increase plasma hydrogen ( ketoacidosis) Stress of infection often precipitates diabetic ketoacidosis

What other long-term complications can develop? Macrovascular: renal artery arteriolosclerosis (stenosis), CAD, PAD Microvascular: retinopathy, nephropathy, neuropathy ( autonomic, impotence) Follow-up tests for diabetes (HbA1C) 6 wk Daily bsl U/A

1. 35yo homosexual male with a 2 week history of nausea, vomiting, diarrhoea. Fever 38.6, hepatosplenomegaly and jaundice. He is a chef and previous IVDU.

Present a differential diagnosis. Acute viral hepatitis D/D: Gastroenteritis, alcohol hepatitis What further history and examination would you like? (Inc: travel history, sexual history, Hep B/C positivity/treatment, HIV status, risk factors for both the blood borne and faecal-oral heps) Investigations FBC, EUC, LFT, stool culture. Blood culture. Coags Serology for the hep virus: Hep A: Ig M of HAV recent infection; ig G past infection Hep B: Hbs Ag 1-6 mth post exposure, HbeAg 1.5-3 mths after acute illness, implies hi infectivity,anti Hbc past infection HbsAg alone: vaccination Hep C: Anti HCV Ab (ig G indicates past infection), PCR,recombinant immunoo blot assay (Western blot) Interpret results: LFTs, INR (typical viral hepatitis picture, transaminases approximately 1000 and included serologies for Hep A, Hep B and Hep C, patient had past HBV, no HCV and Hep A acute infection) How do you know it is acute hepatitis A infection? (talk about IgM rise) Discuss the complications of the hepatitis viruses(Inc. cirrhosis + chronicity for HBV/HCV, hepatocellular cancer for HCV, fulminant liver failure for Hep A- acute liver necrosis) Explain routes of transmission and the various stages of hepatitis infection (ie the difference between a carrier and a chronic). Acute stages, and chronic
Fulminant necrosis Yes Chronic Hepatitis Carrier state HCC No No No Yes Yes (5-10%) yes Yes Yes Yes (>50%) yes Yes Yes No No No

Public health management, treatment of close contacts.


Active immunity: inactivated HAV vaccine Passive immunity: immune serum globulin < 3 month immunity to those at risk ( travellers) Trace back to the source. Report.

1. 60 yo m Case of back pain of 6 weeks

No neurological or abdo symptoms PD/DD Physical examination- what do u look out for? Investigation? PSA is 1200 what could that indicate bony mets?

If wanna confirm dx Wat further ix: DRE, TRUS, biopsy What do they look for in biopsy: gleason score Given xray showed sclerotic vertebrae and calcification in pelvic region What DD I said pagets or other cancer that cause sclerotic lesion

Pathology Exam Questions 1. Lung Cancer, 63yo male presents with two week history haemoptysis and a 2 month history of weight loss. He has smoked 2 packets of cigarettes a day for 45 years. a. Provisional and DDx, does pneumonia usually present with haemoptysis (have to say lobar pneumonia, and has to form abscess/invade blood vessel eg. staph) PDx: bronchogenic carcinoma DDx: TB, metastatic carcinoma, mesothelioma, lobar pneumonia with abscess, pulmonary infarct, bronchiectasis b. Investigations to rule out other causes and confirm your diagnosis? FBC: anaemia, leucocytosis EUC: hypernatraemia with paraneoplastic syndrome CXR: consolidation, pulmonary infarct ABG: evidence of respiratory failure c. What do you look for in sputum? Sputum: microscopy, culture, AFB, cytology d. What are the three types of sampling specimens that can be taken during bronchoscopy and how is the procedure done? Complications of procedure. Bronchoscopy: An instrument (bronchoscope) is inserted into the airways, usually through the nose or mouth, or occasionally through a tracheostomy. This allows the practitioner to examine the patient's airways for abnormalities such as foreign bodies, bleeding, tumors, or inflammation. The instrument is advanced to the trachea and further down into the bronchial system and each area is

inspected as the bronchoscope passes. Specimens may be taken from inside the lungs: biopsies, fluid (bronchoalveolar lavage), or endobronchial brushing. Complications: dysphonia, haemorrhage, pneumothorax, laryngospasm, dyspnoea. e. What types of lung cancer are there? Grading and staging. Investigations for staging. Where in the lung do they arise (bronchi vs pleura and parenchyma vs epithelium)? What symptoms are produced? Small cell: strong association with smoking, rapidly progressive, responds to chemo and radio, high recurrence Non small cell: Squamous cell: closest correlation with smoking, hilar region, more common in men. Often PT hormone secreting tumours hypercalcaemia Adenocarcinoma: more common in women, peripheral mass. Large cell: probably represents poorly differentiated squamous or adenocarcinoma. Grade 1: well differentiated. Grade 2: moderately differentiated. Grade 3: poorly differentiated. Staging: TNM staging system T1-4, N0-3, M0-1 Central: cough, hemoptysis, wheeze, stridor, dyspnoea, pneumonitis Peripheral: pain, cough, dyspnoea, symptoms of lung abscess b. Treatments Surgical resection: wedge resection, segmentectomy, lobectomy, bilobectomy, pneumonectomy. Chemotherapy: mostly for small cell. Radiotherapy: curative or palliative. c. Discuss pot of lung cancer. Bronchial ca squamous cell type.

d. How does lung cancer spread and what organs can it affect? Consequences of invasion. Local spread Pain from chest wall involvement Local invasive to occlude bronchus --> collapse Apical (Pancoast) tumours brachial plexus (pain radiating arm) or sympathetic chain Horners syndrome (ptosis, miosis, anhydrosis) Mediastinal involvement

Recurrent laryngeal nerve hoarseness Superior vena cava syndrome dyspnoea, facial oedema, venous distension of neck veins, headache LN supraclavicular and mediastinal Haematogenous adrenals, bone (pain or hypercalcaemia), liver (capsular pain), brain (headache, confusion, seizures) b. What conditions would affect his prognosis? presence or absence of pulmonary symptoms, tumor size, cell type (histology), degree of spread (stage) and metastases to multiple lymph nodes, and vascular invasion. c. What factors would affect decision to operate? Spirometry may reveal poor respiratory reserve such as in COPD. Dont usually operate on small cell lung cancer. d. Interpret spirometry result. Normal values, type of pattern. How would CAL/COPD affect his chances of surgery? FEV1/FVC normally 0.75 to 0.8. In obstructive disease (asthma, COPD) it is diminished. In restrictive disease (pulmonary fibrosis) are both decreased proportionally so FEV1% may be increased. 1. 19 yo female medical student with chronic bloody diarrhoea with previous episode one year ago. a. What is your provisional diagnosis and what are your differential diagnoses? PDx: inflammatory bowel disease DDx: Gastroenteritis, colorectal carcinoma, pseudomembranous colitis, ischemic colitis b. What further questions would you like to ask her and what would you like to look for on examination? Any recent travel? (infective) Medications? Antibiotics (pseudomembranous colitis) Family history of FAP, HNPCC, IBD AF? ischaemic colitis Urgency, tenesmus, smoking - IBD c. If she was known to have taken antibiotics recently, how might that affect your diagnosis? Consider pseudomembranous colitis C.difficile. Infective cause? d. Peripheral stigmata of IBD, why look for it, what information would it give you? Arthritis Pyoderma gangrenosum Iritis, Episcleritis Erythema nodosum Sclerosing cholangitis Apthous ulcer Clubbing a. Pathology pot of UC: discuss features, rectum doesnt look involved how is this significant?

Colonic pseudopolyps in UC

Rectum not involved suggests prior resection. b. What other investigations would you do now? What would they show you? FBC: iron deficiency anaemia ESR, CRP: raised EUC: dehydration, electrolytes high Stool M/C/S: to exclude infective diarrhoea, C.difficile toxin AXR: to exclude perforation (air under diaphragm), toxic megacolon Colonoscopy+biopsy: UC continuous inflammation of colonic mucosa, oedematous, friable, contact bleeding, pseudopolyps Crohns skip lesions, transmural inflammation with deep fissures, cobblestone appearance. c. If the disease progresses what other complications can take place in the bowel? Colorectal carcinoma Toxic megacolon bowel perforation Haemorrhage 3. Breast cancer, 42 year old woman who discovered a breast lump 15mm in diameter, cousin had breast cancer. a. Ddx Fibrocystic change most breast lumps are benign. Discrete smooth outline. Fibroadenoma mobile breast mouse, well circumscribed. Carcinoma Abscess tender warm cystic Fat necrosis usually following previous trauma or surgery b. Further history and exam which would arouse suspicion Family history Recent rapid enlargement Hard non tender irregular lump Tethering or fixation of the lump Palpable LNs in the axilla Nipple discharge Lymphoedema (including peau dorange) Unilateral visible veins c. Significance of family history and breast cancer. There is an increased risk, however, most breast cancers sporadic.

5-10% of breast cancers are related to inherited mutation eg. P53 first degree relative with breast cancer 1.2-3.0x risk BRAC1 and BRAC2 accounts for a small percentage of familial breast cancer yet those with known mutation has 80% risk. a. What factors determine prognosis? 1. size 2. lymph nodes involved 3. histological type (no special type have worse prognosis) 4. grade of tumour 5. presence of oestrogen and progesterone receptors responsiveness of tumour to anti-oestrogen therapy or oophrectomy or tamoxifen 6. her2 receptor status tumour suppressor gene which increases p27, a protein that halts cell proliferation 7. proliferation rate and degree of aneuploidy 8. lymphovascular invasion a. Discuss features of breast cancer pot.

Invasive ductal carcinoma in fatty breast tissue. IDC starts in the milk ducts and invades the surrounding tissue.

1. Diabetes, Case Protocol 31

a) How would you interpret the history and physical findings? Explain the vital signs - hypovolemic shock. DIABETIC KETOACIDOSIS Thirst blood glucose levels too high for renal resorption glycosuria osmotic diuresis fluid and electrolyte loss Thin accelarated breakdown of fat and muscle due to deficiency of insulin Tachycardia, hypotension hypovolemia Tachypnoea due to acidosis stimulation of chemoreceptors a) What investigations would you perform? Random blood glucose level (single blood glucose >11.1 mmol/L = diabetes) Dipstick heavy ketonuria ABG pH low, base excess negative suggesting acidosis. PaCO2 low indicating some respiratory compensation via hyperventilation. Clinical chemistry a) Interpret ABG results (metabolic acidosis with partial respiratory compensation) pH low, base excess negative suggesting acidosis. PaCO2 low indicating some respiratory compensation via hyperventilation. b) (CLINICAL CHEMISTRY Na decreased, Cl decreased, K increased, Urea Increased, Creatinine Increased, Osmolality increased) Are the glucose and osmolality results diagnostic of DM or do you need to perform more tests? Why is the potassium level so high? What type of renal failure is this? Reduced sodium and chloride due to diuretic sodium loss where this is renal loss of Na and water. Hyperkalaemia shift of potassium from the intracellular space to the extracellular space caused by the acidosis (potassium in exchange for hydrogen ions). Therefore raised levels despite depleted K stores due to diuresis. High urea and creatinine hypovolemia reduced renal blood flow ARF High osmolality due to hyperglycaemia results in water drag into the ECF a) Asked about the pathophysiology of Type 1 Diabetes and DKA. Severe insulin deficiency excessive breakdown of adipose stores increased levels of free fatty acids Oxidation of free fatty acids within in the liver by acetyl CoA produces ketone bodies ketonemia and ketouria If the urinary excretion of ketones is compromised by dehydration, the plasma hydrogen ion concentration increases and systemic metabolic acidosis results. a) What other long-term complications can develop? MACROVASCULAR Coronary artery disease Peripheral vascular disease Stroke MICROVASCULAR

Diabetic nephropathy diffuse glomerulosclerosis, renal arteriosclerosis, pyelonephritis Diabetic neuropathy peripheral neuropathy, mononeuritis multiplex Retinopathy proliferation, dot haemorrhages, exudates a) Follow-up tests for diabetes (HbA1C) Urine tests glucose, ketones Blood glucose HbA1c % circulating haemoglobin with a glucose molecule attached over the life of Hb (~6 weeks) aim for <7% 24 hour microalbuminaemia >20mg/24 hour Fructosamine glycosylated plasma protein 1. 35yo homosexual male with a 2 week history of nausea, vomiting, diarrhoea. Fever 38.6, hepatosplenomegaly and jaundice. He is a chef and previous IVDU. a. Present a differential diagnosis. Acute hepatitis (rare in hep C) hep A: contacts, foreign travel, dietary risk factors (shellfish) hep Bclose contacts, sexual history, blood transfusions, IV drug use, foreign travel, tattoos and body piercing hep E: as for hep A. travel history alcohol: alcohol history drug induced : antibiotics, complementary therapies, Chinese medicine, ecstasy use autoimmune: history of other autoimmune disease b. What further history and examination would you like? (Inc: travel history, sexual history, Hep B/C positivity/treatment, HIV status, risk factors for both the blood borne and faecal-oral heps) c. Investigations LFTs: increased bilirubin, ALT>AST, biliary obstruction: raised GGT, ALP Hep A serology: HAV IgM serology in high titre is diagnostic of acute HAV. IgG confers immunity. Hep B serology HbSAg: viral replication HbeAg: high level of viral replication Anti HbSAg: indicates immunity following infection Anti Hbe: low infectivity in a carrier. Remission but not clearance. Anti HbcIgM: indicates recent infection AntiHbcIgG: indicates exposure to HBV Ultrasound of liver to exclude biliary obstruction and look for chronic liver disease INR, albumin: synthetic function d. Interpret results: LFTs, INR (typical viral hepatitis picture, transaminases approximately 1000 and included serologies for Hep A, Hep B and Hep C,

patient had past HBV, no HCV and Hep A acute infection) How do you know it is acute hepatitis A infection? (talk about IgM rise) e. Discuss the complications of the hepatitis viruses(Inc. cirrhosis + chronicity for HBV/HCV, hepatocellular cancer for HCV, fulminant liver failure for Hep A) Cirrhosis replacement of liver tissue by scar tissue and regenerative nodules (HBV, HCV) Hepatocellular carcinoma, cholangiocarcinoma (HCV) Fulminant liver failure for hep A Portal hypertension Hepatic encephalopathy a. Explain routes of transmission and the various stages of hepatitis infection (ie the difference between a carrier and a chronic). Hepatitis A and E: transmitted by enteric route (faecal oral) Hep B and D: blood products, IV drug users, sexual intercourse, direct contact Hep C: blood, IVDU, acupuncture, sexual and vertical transmission (rare) a. Public health management, treatment of close contacts. Doctors, hospitals and laboratories must confidentially notify cases of hepatitis A infection to the local Public Health Unit. Public Health Unit staff follow special guidelines for managing cases of hepatitis A in people who attend or work at a child care centre, and in people who handle food for sale. Public Health Unit staff also investigate outbreaks of hepatitis A to identify the cause of the outbreak, control its spread and prevent further infections. Normal immunoglobulin (IG) 0.02 mL/kg body weight intramuscularly is recommended for: household and sexual contacts of the case staff and children in close contact with a case in a childcare centre.

Pharmacology Viva Questions


1.

Asthma, 36yo woman presents to ED with tachycardia, tachypnoea, hypotension, peripheral cyanosis, silent chest with past history of asthma. Salbutamol not effective. Her vitals were as follows: Heart rate 150 Blood pressure 90/50 Respiratory rate 36 Normal urinalysis What do you think is happening here?

Respiratory arrest: status asthmaticus Very serious as 1. Tachypnoea (respiratory rate of 26) 2. Hyperinflated chest and tracheal tug 3. Bilateral inspiratory and expiratory wheezes. 4. Wheeze oso due to mucus secretion 5. Pale hypotension 6. Dyspnoea at rest, no conversation

What are your priorities in managing this case? Airway = O2 Makesure no obstruction: breathing intubate Perfusion alright? Standby cardiac arrest/resus/intubation Triage priority Pulse oximetry if < 98% on room air, then supplemental oxygen (CO2) Cardiac monitor IVC= fluids Airway n ventilation Assisted ventilation or intubated CXR priority

1Oxygen 2Brocholdialtero 3SABA + Ipratropium bromide 4If arrst use adrenaline 5Corticosteroid 6Consider theophylline Seretide: SABA+ Fluticasoe Symbicort:Budesonide with eformoterol

How would you stabilise her blood pressure? Adrenaline, fluids

What investigations would you like to do? What would they show you? CXR, ABG, O2 sats FBC, UEC, BSL Peak expiratory flow rates (PEFR) FEV1 Sputum and blood eosinophilia IgE levels Chest roentgenograms (showing hyperinflation) Arterial Blood gas O2 sats, ABG, CXR. Peak flow meter FEV1

ABG: (uncompensated respiratory acidosis type II respiratory failure), interpret the findings, how would the kidneys compensate for the acidosis? Retain bicarb, secrete H+

Oxygen 75 mmHg Carbon dioxide 50 mmHg Bicarbonate 26 (high normal) Base excess -6 (-3 - 3) Oxygen saturation 76%

Long term pharmacological management 3.Antihistamines ( H1 antagonist) Effective in mild asthma cox inhibitor (-) PGD2 & PGF2alpha leukotriene synthase inhibitor (-) LT syn in neut & eosino eg prirpost, zileutin leukotrine receptor anta eg. Montelukast, zafirlukast well-tolerated esp 2-5yo -Relieving : 1. Antimuscarinic eg. Ipratropium (atrovent)

MOA: inhibit the acetylcholine action thru non-selective inhibition of post-synaptic receptor (-) bronchocon (-) muscus secretion (+) mucociliary clearance Not effective against allergen challenge useful as an adjunct to B2 agonist (synergist) mayb of more benefit in COPD than asthma 2. B2 adrenoreceptor agonist eg. Salbutamol, salmeterol MOA: i.thru selective on B2 receptsor but still can affect B1 receptor on myocardium Actions: bronchodilation (-) vagal tone) Increase mucociliary clearance ii.mast cell stabilization as there is B adrenorecptor on mast cell SE: tremors (most common), anxiety CVS: tachycardia, palpitation, cardiac arrhythmia, hypokalimia Serious hypokalemia of used with xanthines & glucocorticoids Worsen glucose tolerance in diabetics trigger gluconeogenies May have paradoxical bronchospasm Administration Salbutamol thru inhalation (aerosol, powder, nebulizer) oso can give po or iv Acting w/i minutes Long acting can help nocturnal symptoms and reduce morning dips. 3.Glucocortioids eg prednisolone Multiple mechanism to -Relax bronchospasm (but do not cuase bronchodilation) -decrease mucus secretion -antagonize histamine action -Inhibit Ab 4mation -inhibit phospholipase A2 Blk both early & late response Administration: Act over days. Best inhaled eg. Beclometasone via spacer (or powder) Mayb given PO or iv. oral steroids r used acutely in hi dose short course and longer term in lower dose if ctrl is not optimal on inhalers SE:

-oropharyngeal candidiasis in the immunocom (dirty inhaler) Rincse mouth afer inhlation -Hi dose hypothalamic pituitary suppression adrenal suppression (retarded growth in kids) -Dysphonia (weakening of adductor mus in vocal cord). .Xanthines Relax bronchial smooth mus MOA adenosine antagonist adenosine is a vasodilator and bronchoconstrictor xanthine inhibit adenylcyclase cAMP [Ca]i inhibit contractile phosphodiesterase inhibitor prevent cAMP breakdown

other effect generalized smooth mus relax mast cell stabilization improve diaphragmatic functions as increase catcholamine rel SE: small therapeutic index (so not first choice) If in toxic range will cause CNS: seizure (kid), tremor, insomnia, alertness CVS, ardiac dysrrythmia, hypotension, hypokalemia GI upset So check theophylline lvl and do ECG and plasma lvl monitoring after 24 hr if iv therapy is used. Administration: po. Useful as an adjunct if inhaled therapy is inadequate. In acute severe asthma, given IV Oso try as prophylaxis po to prevent morning dipping Eg. Theophylline

How would you monitor her asthma? Daily Peak flow meter by herself Spirometry in clinic Amt of attack

1. IBD, 43yo female presents with two week history of bloody diarrhoea, 12 times in last 10 hrs, pain in left and right upper quadrants, previous episode 2 yrs ago that resolved. Provisional and DDx? PDx: UC/CD If you say gastroenteritis, they will ask what organisms are responsible. DD: Bloody diarrhea: shigella, campylobacter, salmonella CRC How would you investigate? Colonoscope, stool culture ESR, CRP, CXR, FBC, ultrasound of right upper quadrant UC was previously Dxed. An endoscopy was performed this time again. What features would point to the Dx of UC? Bloody diarrhea, recurrent episodes. How would you distinguish it from your differentials? Pseudopoly of the colon, extra intestinal manifestation: uveitis, arthritis.AS, clubbing 1. 2. 3. 4. Hyperaemia, bloody Granular appearance, continuous, sharp junction to normal Superficial ulcers Loss of haustrations eventually

Complications of UC Epithelial dysplasia carcinoma Toxic megacolon, perforation, peritonitis Septic shock Dehydration, hemorrhage anemia Last time, her episode resolved with prednisolone and sulfasalasine. What are the mechanisms of action of these 2 drugs at the molecular level? Side effects. Would you use aziothioprine?

Prednisolone : reduce inflammatory cytokindes by reduce immune response Sulfasalazine immune modulator reduce pdtn of eicosanoid and other cytokindes and prostaglandin Sulfasalazine possesses both antiinflammatory (5-ASA) and antibacterial (sulfapyridine) properties 5 ASA poorly absorb so stay in colon This time, she didn't respond to these 2 drugs. What additional medications would you use? Would you use any biological agents? Will use aziothioprine first if doesnt respond to steroid If doesnt respond to aziothioprine then use Methotrexate, TNF inhibitor infliximab or surgical resection 1. The management is directed towards both symptomatic improvement and controlling the disease process 2. i) Pharmacological Approach 3. The principle of drug treatment are based upon treatment of active disease and prevention of relapse

1. Anaphylaxis, patient presents with a bee sting. Had been stung before but had only a mild response. Now presents with tachycardia, tachypnoea, pale, drowsy, angioedema and insp and exp stridor. Provisional dx Anaphylactic shock Investigations FBC, UEC, ABG, O2 sats, Ig E ABC ( including hi-flow O2) Raise foot of the bed IV access X w (wide bore; get help if this takes > 2min) Identify and treat underlying cause Infuse crystalloid fast to raise BP (unless cardiogenic shock) Seek expert help early

Interpret ABG (mixed resp and metabolic acidosis) and EUC (renal failure, electrolyte disturbance) Provide pathophysiology behind all clinical manifestations Vasodilation peripheral. Decrease renal perfusion ARF metabolic acidosis Upper airway constriction from bronchospasms. Respiratory acidosis Pale, drowsy and incoherent = shock with perfusion to brain. in Swollen face, eyes, tongue = allergic reaction peripheral vasodilatation vascular leakage oedema. Urticaria = Wheal oedema; Flare hyperaemia. Stridor = pharyngeal and laryngeal oedema upper airway obstruction. Bilateral wheezes/rhonchi = anaphylactic response bronchospasm, mucus secretion, bronchiole constriction bronchiolar obstruction. Tachypnoea = bodys compensatory response to airway obstruction by RR.

Explain why patient has reacted with anaphylaxis only now and not previously Delayed hypersensitivity type 1 Need to develop memory B cell Involve IgE activation mast cell degranulation etc. E.g. food (salmon), beestings, drugs (penicillin).

Explain mechanism of adrenaline

1.

Osteoporosis, post-menopausal lady with fractured NOF and osteoporosis What are T and Z scores on bone mineral density and cut offs for osteoporosis and osteopenia.

T score to teenage Z score to age equivalent Osteopenia is -12.5 Osteoperosos is < -2.5

Secondary osteoproroisis Bone building blk Ca, PO34-, Vit D(1,25 Vit d) Mm Fbc/esr/euc Bj URINE Immunofixation of paraptn

Hyperparathyroidism PTH Hyperthryoridism TFT Hypogonadism Testosterone, Gonadotrophin

Screening blood tests patient may require. Vitamin d, calcium, phosphate FBC, ESR, UEC, BJ urine, immunofixation of paraptn, PTH, TFT, testosterone,gonadotrophin Osteocalcin(OC) Bone specific alkaline phosphatase Other: PSA DEXA Pharmacological management Diet Wt bearing exercise,strengthen muscle and lig too sun exposure Supplements: Ca, vit d

-HRT for post menopausal women continuous estrogen-progestin therapy does not protect against cardiovascular disease, and it increases the risk of invasive breast cancer, coronary heart disease, stroke, thromboembolic disease, and dementia. Likewise, unopposed estrogen does not appear to be cardioprotective and is also associated with an increased risk of stroke, thromboembolic disease, and dementia. As a result of this evidence, long-term HRT is no longer recommended for the disease prevention Most drug therapies work by decreasing bone resorbption

-biphosphonates such as risedronate that act an inhibitor of osteoclastic activity. -Raloxifene is a selective estrogen receptor modulator (SERM) include decreased risk for uterus cancer, because raloxifene acts antagonistically to estrogen on the uterus. Prevention HRT w/I 2 years of menopause Calcium supplement (except if taking calcitriol) Weight bearing exercise Lifestyle: stop smoking, limit alcohol and caffeine Vitamind and sunlight Avoid fall eg sedative medication Adequate nutritions Goal is to reduce further loss. -Calcitriol vit D metabolite(Rocaltrol) Used for failed HRT Useful for corticosteroid induced osteoporosis, aovid calcium supplements during therapy -Biphosphonate( decrease bone absorption) give if ca and vit D level is normal. Limited to use up to 5 years as lack of research beyond that. Side effects: gastric duodenal ulcer, osteonecrosi of jaw Alendronate (Fosamax), Etidronate, Risedronte (Actonel). -Strontium ranelate( protos) prevent further loss, strengthen the bone -Raloxifene (SERM, selective estrogen-receptor modulator) used as prevention of osteoporosis in PMS women. Has estrogenic actions on bone and antiestrogenic action on uterus and breast. It is used as second line due to lack of studies (eg. Hip fracture)

Benefits of HRT for this woman (depends if she is experiencing any symptoms of menopause)

1. Pain management, man who recently had abdominal surgery. Review his medication chart and comment on what is wrong with it. He is receiving morphine IM, paracetamol oral and odancetron. Hes receiving morphine PRN, why is this wrong? PRN is for breakthrough pain on top of regular pain relief. As he is a post sx pt, he needs regular pain relief How

should it be administered? Need to mention patient-controlled analgesia. Or iv infusion. Hes receiving morphine IM, why is this bad? Erratic absorption. Slower effect and harder to titrate according to pts response Hes on oral paracetamol, not good if he just had abdominal surgery. Side effects of morphine Cardio and respi depression, sedation, constipation, N + V What is odancetron? Why is it being given? Serotonin receptor antagonist. Inhibit vomit centre in the brain

If he was an IVDU, how would you alter his pain management? Titrate the dose carefully, prevent under treatment. Often need higher dosage as dvp dependence At the mean time, d/c plan will be metadonne.

The Ladder Bottom rung of ladder (mild pain): Non opioid +/- adjuvant Next rung of ladder (worse pain): Weaker opioid +/- non opioid +/- adjuvant Highest rung of ladder (worst pain): Strong opioid +/- non opioid +/- adjuv The pain ladder is a term originally coined by the World Health Organisation (WHO) to describe a method of controlling pain[1]. It was originally described for the management of cancer pain, but is now widely used by medical professionals for the management of all types of pain. The general principle in pain management is to start at the bottom rung of the ladder, and then to climb the ladder if pain is still present. The medications range from household, over the counter drugs with minimal side-effects at the lowest rung to powerful opioids. Opioid usage carries a high risk of addiction and adverse effects if abused. 6.Pc: epigastric pain + hematemesis Chronic epigastric pain of several years, relieve by food + antacids PD/DD So is it gastric or duodenal : gastric as relieve by food Blood test/investigation in acute setting endoscopy What kind of pathology will give the same blood picture of anemia Endoscopy shows gastric ulcer with hpylori Treatment in acute setting If keep bleeding, what can u do? Cauterization or injection of epinephrine, ligation What kind of therapy for H.pylori?Triple therapy: PPI+ amoxicillin+ clindamycin +/metronidazole

Other options? If cant use PPI or antibiotics? If given treatment, What kind of follow up How do u test for in follow up setting

Pharmacology Viva Questions 1. Asthma, 36yo woman presents to ED with tachycardia, tachypnoea, hypotension, peripheral cyanosis, silent chest with past history of asthma. Salbutamol not effective. Her vitals were as follows: Heart rate 150 Blood pressure 90/50 Respiratory rate 36 Normal urinalysis a. What do you think is happening here? Acute asthma attack tachycardia >110, tachypnoea >25, silent chest, salbutamol not effective, pulsus paradoxus >20mmHg (exaggerated drop in BP during inspiration because of the more negative intrathoracic pressures needed to breath at high lung volumes), Peak flow rate <50% predicted b. What are your priorities in managing this case? Priorities in acute asthma management: to prevent death, to restore and maintain optimal pulmonary function, prevent early relapse Oxygen therapy by tight fitting facemask Nebulised salbutamol + prednisolone CXR to exclude pneumothorax chest asymmetry, reduced air entry, tracheal deviation Start glucocorticoid therapy IV hydrocortisone or oral prednisolone Urgent blood gas Consider IV aminophylline if life threatening after 15-30 mins. a. How would you stabilise her blood pressure?

b. a.

What investigations would you like to do? What would they show you? CXR: look for pneumothorax Arterial blood gases Sputum culture BP: pulsus paradoxus >20mmHg Spirometry: peak flow rate <50% of predicted ABG: interpret the findings, how would the kidneys compensate for the acidosis?

Oxygen 75 mmHg Carbon dioxide 50 mmHg Bicarbonate 26 (high normal) Base excess -6 (-3 - 3) Oxygen saturation 76% uncompensated respiratory acidosis type II respiratory failure (hypoxia with hypercapnoea)=caused by alveolar hypoventilation Compensation of respiratory acidosis: kidneys increase excretion of H+ ion so the urine becomes acidic and thereby increases the retention or reabsorption of HCO3- into the blood. a. Long term pharmacological management 1. Bronchodilators reliever medication adrenergic agonists Dilate bronchial smooth muscle by direct action on 2 adrenergic receptors and inhibit the release of inflammatory mediators by mast cells. Short acting (4-6hrs): Salbutamol and Terbutaline Long acting (12 hrs): Salmeterol SE: tremor and tachycardia Muscarinic receptor antagonists: Ipratropium Bromide Antagonises bronchial constriction caused by parasympathetic stimulation of M1 and M3 receptors when binding Ach. Onset of action is slow(30-60mins) so used in combination with 2 adrenoreceptor agonist. SE: tachycardia, nausea, vomiting, dry mouth Xanthines - Theophylline Inhibits phosphodiesterase (which metabolises cAMP) thus increased intracellular cAMP which could inhibit the activation of inflammatory cells relax smooth muscle Used when 2 agonists ineffective SE: CNS stimulation, cardiac stimulation, nausea, anorexia 2. Symptom controllers Long acting B2 adrenoceptor agonists 3. Anti-inflammatory drugs Glucocorticoids Inhaled glucocorticoids: Beclomethasone, Beudesnoide, Fluticasone Anti-inflammatory (inhibit induction of COX-2) and immunosuppressant (inhibit influx of inflammatory cells in lung) SE: Oral thrush and hoarseness Suppression of immune system

Systemic effect bruising, dermal thinning, adrenal suppression, altered bone metabolism osteoporosis Impaired wound healing Development of diabetes and peptic ulcers Cushings syndrome 4. Anti-tussives: opiods Codeine, Dextromethorphan, Pholcodine Should only be used if unproductive or excessive as cough removes foreign material and mucous secretions from the bronchi 5. Acute exacerbations Antibiotics, O2 therapy, bronchodilators and glucocorticoids

a. How would you monitor her asthma? Take history and questionnaire Expiratory peak flows before and after salbutamol Spirometry annually 1. IBD, 43yo female presents with two week history of bloody diarrhoea, 12 times in last 10 hrs, pain in left and right upper quadrants, previous episode 2 yrs ago that resolved. a. Provisional and DDx? If you say gastroenteritis, they will ask what organisms are responsible. PDx: inflammatory bowel disease DDx gastroenteritis Salmonella, Shigella, Campylobacter, E.Coli, Entamoeba histolytica, Giardia lamblia Colorectal cancer Colonic polyps Pseudomembranous colitis Ischaemic colitis b. How would you investigate? FBC: iron deficiency anaemia ESR, CRP: raised EUC: dehydration, electrolytes high Stool M/C/S: to exclude infective diarrhoea, C.difficile toxin AXR: to exclude perforation (air under diaphragm), toxic megacolon Colonoscopy+biopsy: UC continuous inflammation of colonic mucosa, oedematous, friable, contact bleeding, pseudopolyps Crohns skip lesions, transmural inflammation with deep fissures, cobblestone appearance. UC was previously Dxed. An endoscopy was performed this time again. c. What features would point to the Dx of UC? How would you distinguish it youre your differentials?

UC continuous inflammation of colonic mucosa, oedematous, friable, contact bleeding, pseudopolyps d. Complications of UC Endoscopic image of ulcerative colitis affecting the left side of the colon. The image shows confluent superficial ulceration, and loss of mucosal architecture.

Last time, her episode resolved with prednisolone and sulfasalasine (aminosalicylate). e. What are the mechanisms of action of these 2 drugs at the molecular level? Side effects. Would you use aziothioprine? Prednisolone crosses cell membranes and binds with high affinity to specific cytoplasmic receptors. The result includes inhibition of leukocyte infiltration at the site of inflammation, interference in the function of mediators of inflammatory response, suppression of humoral immune responses, and reduction in edema or scar tissue. The anti-inflammatory actions of corticosteroids are thought to involve phospholipase A2 inhibitory proteins, lipocortins, which control the biosynthesis of potent mediators of inflammation such as prostaglandins and leukotrienes. Sulfasalazine: anti-inflammatory inhibits lipoxygenase, prostaglandins and leukotrienes. Impairs folate absorption. The most common side effects of sulfasalazine are headache, nausea, fever, rash, and reversible infertility in men. Azathioprine (immunosuppressive) SE: suppresses immune system, bone marrow suppression, hepatitis, alopecia, skin rash f. This time, she didn't respond to these 2 drugs. What additional medications would you use? Would you use any biological agents? Other medications for UC: Corticosteroids for remission eg. hydrocortisone TNF inhibitors eg. Infliximab Ciclosporin for severe acute colitis 1. Anaphylaxis, patient presents with a bee sting. Had been stung before but had only a mild response. Now presents with tachycardia, tachypnoea, pale, drowsy, angioedema and insp and exp stridor.

a. Provisional dx Anaphylactic shock DDx: airway obstruction (status asthmaticus, foreign body, acute bacterial epiglossitis) Angioedema Vasovagal syncope Globus hystericus b. Investigations Airway maintenance look for obstruction from swollen tongue, stridor Pulse tachycardic Blood pressure hypotension ECG must be done before administration of adrenaline FBC eosinophilia Skin prick tests intradermal injection of a small dose of drug ABG mixed resp and metabolic acidosis EUC renal failure, electrolyte disturbance Blood test tryptase released from mast cells c. Interpret ABG (mixed resp and metabolic acidosis) and EUC (renal failure, electrolyte disturbance) Metabolic acidosis Widespread vasodilation decreased perfusion cellular ischemia switch from aerobic to anaerobic metabolism metabolic acidosis Respiratory acidosis Pharyngeal oedema airway obstruction inability to blow off CO2 respiratory acidosis d. Provide pathophysiology behind all clinical manifestations Pale, drowsy shock with decreased perfusion to the brain Swollen face, eyes, tongue allergic reaction peripheral vasodilation increased vascular leakage oedema Stridor pharyngeal and laryngeal oedema upper airway obstruction Bilateral wheeze bronchospasm, mucus secretion, bronchiole constriction e. Explain why patient has reacted with anaphylaxis only now and not previously Allergic hypersensitivity (type 1) Mediated by IgE antibodies formed in response to a particular allergen/antigen. These are bound to mast cells and basophils. Normally the function of IgE is to fight infections. In sensitised individuals, they respond by synthesising large amounts of IgE to an allergen. On re-exposure to the same allergen, the allergen binds to and cross links with the IgE on mast cells, resulting in degranulation and release of performed inflammatory mediators, and de novo synthesis of other mediators.

Histamine bronchial smooth mm contraction and increased vascular permeability Adenosine enhances mast cells mediator release and causes bronchoconstriction Leukotrienes more potent than histamine PAF cause platelet aggregation, release of histamine, bronchospasm, increased vascular permeability and vasodilation. a. Explain mechanism of adrenaline Adrenaline stimulates: adrenoreceptors to increase peripheral vascular resistance thus improving BP and coronary perfusion, reverse peripheral vasodilation. 1 adrenoreceptors for positive inotropic and chronotropic cardiac effects. 2 adrenoreceptors bronchodilation and increase cAMP in mast cells to reduce release of inflammatory mediators. 1. Pain management, man who recently had abdominal surgery. a. Review his medication chart and comment on what is wrong with it. He is receiving morphine IM, paracetamol oral and odancetron. b. Hes receiving morphine PRN, why is this wrong? How should it be administered? Need to mention patient-controlled analgesia. c. Hes receiving morphine IM, why is this bad? d. Hes on oral paracetamol, not good if he just had abdominal surgery. e. Side effects of morphine f. What is odancetron? Why is it being given? g. If he was an IVDU, how would you alter his pain management?

Pharmacology Viva Questions


1.

Asthma, 36yo woman presents to ED with tachycardia, tachypnoea, hypotension, peripheral cyanosis, silent chest with past history of asthma. Salbutamol not effective. Her vitals were as follows: Heart rate 150 Blood pressure 90/50 Respiratory rate 36 Normal urinalysis What do you think is happening here? Respiratory arrest: status asthmaticus Very serious as

1. Tachypnoea (respiratory rate of 26) 2. Hyperinflated chest and tracheal tug

3. Bilateral inspiratory and expiratory wheezes. 4. Wheeze oso due to mucus secretion 5. Pale hypotension 6. Dyspnoea at rest, no conversation

What are your priorities in managing this case? Airway = O2 Makesure no obstruction: breathing intubate Perfusion alright? Standby cardiac arrest/resus/intubation Triage priority Pulse oximetry if < 98% on room air, then supplemental oxygen (CO2) Cardiac monitor IVC= fluids Airway n ventilation Assisted ventilation or intubated CXR priority

1Oxygen 2Brocholdialtero 3SABA + Ipratropium bromide 4If arrst use adrenaline 5Corticosteroid 6Consider theophylline Seretide: SABA+ Fluticasoe Symbicort:Budesonide with eformoterol

How would you stabilise her blood pressure? Adrenaline, fluids What investigations would you like to do? What would they show you? CXR, ABG, O2 sats FBC, UEC, BSL

Peak expiratory flow rates (PEFR) FEV1 Sputum and blood eosinophilia IgE levels Chest roentgenograms (showing hyperinflation) Arterial Blood gas O2 sats, ABG, CXR. Peak flow meter FEV1

ABG: (uncompensated respiratory acidosis type II respiratory failure), interpret the findings, how would the kidneys compensate for the acidosis? Retain bicarb, secrete H+

Oxygen 75 mmHg Carbon dioxide 50 mmHg Bicarbonate 26 (high normal) Base excess -6 (-3 - 3) Oxygen saturation 76%

Long term pharmacological management 3.Antihistamines ( H1 antagonist) Effective in mild asthma cox inhibitor (-) PGD2 & PGF2alpha leukotriene synthase inhibitor (-) LT syn in neut & eosino eg prirpost, zileutin leukotrine receptor anta eg. Montelukast, zafirlukast well-tolerated esp 2-5yo -Relieving : 1. Antimuscarinic eg. Ipratropium (atrovent) MOA: inhibit the acetylcholine action thru non-selective inhibition of post-synaptic receptor (-) bronchocon (-) muscus secretion (+) mucociliary clearance

Not effective against allergen challenge useful as an adjunct to B2 agonist (synergist) mayb of more benefit in COPD than asthma 2. B2 adrenoreceptor agonist eg. Salbutamol, salmeterol MOA: i.thru selective on B2 receptsor but still can affect B1 receptor on myocardium Actions: bronchodilation (-) vagal tone) Increase mucociliary clearance ii.mast cell stabilization as there is B adrenorecptor on mast cell SE: tremors (most common), anxiety CVS: tachycardia, palpitation, cardiac arrhythmia, hypokalimia Serious hypokalemia of used with xanthines & glucocorticoids Worsen glucose tolerance in diabetics trigger gluconeogenies May have paradoxical bronchospasm Administration Salbutamol thru inhalation (aerosol, powder, nebulizer) oso can give po or iv Acting w/i minutes Long acting can help nocturnal symptoms and reduce morning dips. 3.Glucocortioids eg prednisolone Multiple mechanism to -Relax bronchospasm (but do not cuase bronchodilation) -decrease mucus secretion -antagonize histamine action -Inhibit Ab 4mation -inhibit phospholipase A2 Blk both early & late response Administration: Act over days. Best inhaled eg. Beclometasone via spacer (or powder) Mayb given PO or iv. oral steroids r used acutely in hi dose short course and longer term in lower dose if ctrl is not optimal on inhalers SE: -oropharyngeal candidiasis in the immunocom (dirty inhaler) Rincse mouth afer inhlation -Hi dose hypothalamic pituitary suppression adrenal suppression (retarded growth in kids) -Dysphonia (weakening of adductor mus in vocal cord).

.Xanthines Relax bronchial smooth mus MOA adenosine antagonist adenosine is a vasodilator and bronchoconstrictor xanthine inhibit adenylcyclase cAMP [Ca]i inhibit contractile phosphodiesterase inhibitor prevent cAMP breakdown

other effect generalized smooth mus relax mast cell stabilization improve diaphragmatic functions as increase catcholamine rel SE: small therapeutic index (so not first choice) If in toxic range will cause CNS: seizure (kid), tremor, insomnia, alertness CVS, ardiac dysrrythmia, hypotension, hypokalemia GI upset So check theophylline lvl and do ECG and plasma lvl monitoring after 24 hr if iv therapy is used. Administration: po. Useful as an adjunct if inhaled therapy is inadequate. In acute severe asthma, given IV Oso try as prophylaxis po to prevent morning dipping Eg. Theophylline

How would you monitor her asthma? Daily Peak flow meter by herself Spirometry in clinic Amt of attack 1. IBD, 43yo female presents with two week history of bloody diarrhoea, 12 times in last 10 hrs, pain in left and right upper quadrants, previous episode 2 yrs ago that resolved. Provisional and DDx? PDx: UC/CD If you say gastroenteritis, they will ask what organisms are responsible.

DD: Bloody diarrhea: shigella, campylobacter, salmonella CRC How would you investigate? Colonoscope, stool culture ESR, CRP, CXR, FBC, ultrasound of right upper quadrant UC was previously Dxed. An endoscopy was performed this time again. What features would point to the Dx of UC? Bloody diarrhea, recurrent episodes. How would you distinguish it from your differentials? Pseudopoly of the colon, extra intestinal manifestation: uveitis, arthritis.AS, clubbing 1. 2. 3. 4. Hyperaemia, bloody Granular appearance, continuous, sharp junction to normal Superficial ulcers Loss of haustrations eventually

Complications of UC Epithelial dysplasia carcinoma Toxic megacolon, perforation, peritonitis Septic shock Dehydration, hemorrhage anemia Last time, her episode resolved with prednisolone and sulfasalasine. What are the mechanisms of action of these 2 drugs at the molecular level? Side effects. Would you use aziothioprine? Prednisolone : reduce inflammatory cytokindes by reduce immune response Sulfasalazine immune modulator reduce pdtn of eicosanoid and other cytokindes and prostaglandin Sulfasalazine possesses both antiinflammatory (5-ASA) and antibacterial (sulfapyridine) properties 5 ASA poorly absorb so stay in colon

This time, she didn't respond to these 2 drugs. What additional medications would you use? Would you use any biological agents? Will use aziothioprine first if doesnt respond to steroid

If doesnt respond to aziothioprine then use Methotrexate, TNF inhibitor infliximab or surgical resection 1. The management is directed towards both symptomatic improvement and controlling the disease process 2. i) Pharmacological Approach 3. The principle of drug treatment are based upon treatment of active disease and prevention of relapse

1. Anaphylaxis, patient presents with a bee sting. Had been stung before but had only a mild response. Now presents with tachycardia, tachypnoea, pale, drowsy, angioedema and insp and exp stridor. Provisional dx Anaphylactic shock Investigations FBC, UEC, ABG, O2 sats, Ig E ABC ( including hi-flow O2) Raise foot of the bed IV access X w (wide bore; get help if this takes > 2min) Identify and treat underlying cause Infuse crystalloid fast to raise BP (unless cardiogenic shock) Seek expert help early

Interpret ABG (mixed resp and metabolic acidosis) and EUC (renal failure, electrolyte disturbance) Provide pathophysiology behind all clinical manifestations Vasodilation peripheral. Decrease renal perfusion ARF metabolic acidosis Upper airway constriction from bronchospasms. Respiratory acidosis Pale, drowsy and incoherent = shock with in perfusion to brain.

Swollen face, eyes, tongue = allergic reaction peripheral vasodilatation vascular leakage oedema. Urticaria = Wheal oedema; Flare hyperaemia. Stridor = pharyngeal and laryngeal oedema upper airway obstruction. Bilateral wheezes/rhonchi = anaphylactic response bronchospasm, mucus secretion, bronchiole constriction bronchiolar obstruction. Tachypnoea = bodys compensatory response to airway obstruction by RR.

Explain why patient has reacted with anaphylaxis only now and not previously Delayed hypersensitivity type 1 Need to develop memory B cell Involve IgE activation mast cell degranulation etc. E.g. food (salmon), beestings, drugs (penicillin).

Explain mechanism of adrenaline

Pharmacological management Diet Wt bearing exercise,strengthen muscle and lig too sun exposure Supplements: Ca, vit d

-HRT for post menopausal women continuous estrogen-progestin therapy does not protect against cardiovascular disease, and it increases the risk of invasive breast cancer, coronary heart disease, stroke, thromboembolic disease, and dementia. Likewise, unopposed estrogen does not appear to be cardioprotective and is also associated with an increased risk of stroke, thromboembolic disease, and dementia. As a result of this evidence, long-term HRT is no longer recommended for the disease prevention Most drug therapies work by decreasing bone resorbption

-biphosphonates such as risedronate that act an inhibitor of osteoclastic activity. -Raloxifene is a selective estrogen receptor modulator (SERM) include decreased risk for uterus cancer, because raloxifene acts antagonistically to estrogen on the uterus. Prevention HRT w/I 2 years of menopause Calcium supplement (except if taking calcitriol) Weight bearing exercise Lifestyle: stop smoking, limit alcohol and caffeine Vitamind and sunlight Avoid fall eg sedative medication Adequate nutritions Goal is to reduce further loss. -Calcitriol vit D metabolite(Rocaltrol) Used for failed HRT Useful for corticosteroid induced osteoporosis, aovid calcium supplements during therapy -Biphosphonate( decrease bone absorption) give if ca and vit D level is normal. Limited to use up to 5 years as lack of research beyond that. Side effects: gastric duodenal ulcer, osteonecrosi of jaw Alendronate (Fosamax), Etidronate, Risedronte (Actonel). -Strontium ranelate( protos) prevent further loss, strengthen the bone -Raloxifene (SERM, selective estrogen-receptor modulator) used as prevention of osteoporosis in PMS women. Has estrogenic actions on bone and antiestrogenic action on uterus and breast. It is used as second line due to lack of studies (eg. Hip fracture)

Benefits of HRT for this woman (depends if she is experiencing any symptoms of menopause)
1. Osteoporosis, post-menopausal lady with fractured NOF and osteoporosis a. What are T and Z scores on bone mineral density and cut offs for osteoporosis and osteopenia. Bone mineral density tested with dual energy X-ray absorptiometry. T score: compared to healthy young Caucasian women Z score: compared to age matched control Osteoporosis: >2.5 SD below the mean

Osteopenia: 1-2.5 SD below b. Asked about medical conditions related to osteoporosis. Hormone deficiencies: Turners, Kleinfelters, oophrectomy, premature ovarian failure Endocrine disease: Cushings syndrome, hyperparathyroidism, thyrotoxicosis, diabetes Malnutrition/malabsorption: lack of calcium absorption Crohns disease, celiac disease, lactose intolerance Rheumatological disease (due to disease and corticosteroid Rx): RA, ankylosing spondylitis, SLE Renal insufficiency: can lead to osteodystrophy Haematological: multiple myeloma, monoclonal gammopathy, lymphoma, leukemia a. Screening blood tests patient may require. FBC, blood film, serum and urinary EPG multiple myeloma EUC renal osteodystrophy Calcium, phosphate, Alk phosphatase bone mets, osteomalacia, Paget, fractures Serum PTH and Vit D renal osteodystrophy, osteomalacia a. Pharmacological management Bisphosphonates: become incorporated within the bone matrix. When bone is ingested by osteoclasts, the drug is released within the cell to cause osteoclast death, thereby inhibiting bone resorption. Eg. Alendronate, Risendronate Raloxifene: a selective oestrogen receptor modulator acts as an oestrogen receptor agonist in some tissue and antagonist in others. Increase bone mass Calcitonin: slows bone loss by opposing the effects of PTH. Calcium and VitD supplements HRT a. Benefits of HRT for this woman (depends if she is experiencing any symptoms of menopause) Not recommended unless there are other indications for its use. HRT is effective in treating menopausal symptoms and preventing osteoporotic fractures. Increased risk of breast cancer, endometrial cancer and venous thromboembolism.

1. Pain management, man who recently had abdominal surgery. Review his medication chart and comment on what is wrong with it. He is receiving morphine IM, paracetamol oral and odancetron.

Hes receiving morphine PRN, why is this wrong? PRN is for breakthrough pain on top of regular pain relief. As he is a post sx pt, he needs regular pain relief How should it be administered? Need to mention patient-controlled analgesia. Or iv infusion. Hes receiving morphine IM, why is this bad? Erratic absorption. Slower effect and harder to titrate according to pts response Hes on oral paracetamol, not good if he just had abdominal surgery. Side effects of morphine Cardio and respi depression, sedation, constipation, N + V What is odancetron? Why is it being given? Serotonin receptor antagonist. Inhibit vomit centre in the brain

If he was an IVDU, how would you alter his pain management? Titrate the dose carefully, prevent under treatment. Often need higher dosage as dvp dependence At the mean time, d/c plan will be metadonne.

The Ladder Bottom rung of ladder (mild pain): Non opioid +/- adjuvant Next rung of ladder (worse pain): Weaker opioid +/- non opioid +/- adjuvant Highest rung of ladder (worst pain): Strong opioid +/- non opioid +/- adjuv The pain ladder is a term originally coined by the World Health Organisation (WHO) to describe a method of controlling pain[1]. It was originally described for the management of cancer pain, but is now widely used by medical professionals for the management of all types of pain. The general principle in pain management is to start at the bottom rung of the ladder, and then to climb the ladder if pain is still present. The medications range from household, over the counter drugs with minimal side-effects at the lowest rung to powerful opioids. Opioid usage carries a high risk of addiction and adverse effects if abused. 6.Pc: epigastric pain + hematemesis Chronic epigastric pain of several years, relieve by food + antacids PD/DD So is it gastric or duodenal : gastric as relieve by food Blood test/investigation in acute setting endoscopy What kind of pathology will give the same blood picture of anemia Endoscopy shows gastric ulcer with hpylori Treatment in acute setting

If keep bleeding, what can u do? Cauterization or injection of epinephrine, ligation What kind of therapy for H.pylori?Triple therapy: PPI+ amoxicillin+ clindamycin +/metronidazole Other options? If cant use PPI or antibiotics? If given treatment, What kind of follow up How do u test for in follow up setting

Station A: Much the same as case protocol 14, 65 man with thoracic back pain, distended bladder. PSA raised. 1. Present DDx: (include primary and secondary disorders, including primary malignancy, trauma, fracture, including secondary malignancies, including prostate, lung, kidney, colon) 2. What investigations are appropriate? (Inc. specifics of how to order the MRI - full spine lateral MRI required) 2a. Have a look at this: result of PSA 2b. (I think my examiner was just being a prick here) Discuss public health perspectives of PSA screening. How 'exactly' would you describe risks and benefits to the patient? [He cut me off once I started to give him my standard spiel anyway] 3. Discussion of anatomy of prostate: inc. zones 4. How does prostate cancer spread? (inc. discussion of prostate --> intervertebral venous plexus, including structure and location of intervertebral venous plexuses)

(1) basic prostate anatomyIts a fibromuscular glandular organ that surrounds the prostatic urethra (other parts of urethra being membranous and spongiose distally- look it up if u want to kno details). the prostatic urethra recieves the ejaculatory ducts at the prostatic utricle, and the prostatic glands drain lateral to this in the prostatic sinus (look up a picture). Its surrounded by a capsule, outside of which is the prostatic venous plexus. Outside this is the visceral pelvic fascia that encloses BOTH bladder and prostate together (i.e. the prostatic venous plexus and vesicle veins (of bladder) are in continuity). These are THIN walled, VALVELESS veins- importance of this comes later. I am quite sure the anatomy is NOT about central and peripheral zonesthat is more an ultrasonographers or radiologists textbooks- an anatomist (ie.Vu) would like to hear about the anterior, median, posterior and lateral (left and right) lobes of the prostate. Anterior relations: basically there is a pubic syphysis in front of it, and fat in between the sympysis and the gland, the area of fat is called the retropubic space- (eponymous name is space of Retzius- use it because it was in the campus day notes). Behind it is rectovescical

fascial septum (fascia of Denonvillier, not in notes). Laterally is the levator ani muscle (know what its made up of). (2). NPH. The enlargement of the prostate is mainly in the medial, lateral lobes and a bit of posterior also. The medial lobe expands UPWARDS into the bladder. Below the prostate is the urethral sphincter, and above it is the vesical sphincter, and the vesical sphincter is encroached upon, leading to leakage of urine into the prostatic urethra and an intense desire to micturate (urgency). The enlarged median lobe can also compress the urethral orifice in the bladder and act like a flap valve everytime the pressure (to pee) rises, leading to difficulty starting a stream (hesitancy). When the stream starts, there is poor stream because the prostatic urethra is elongated, compressed and distorted by the expanding medial and lateral lobes. Lastly, the median lobe which expanded upwards can create a bump/elevation in the bladder trigone and this is called the prostatic uvula- its important because there can be stagnant urine posterior to this which cant get out, leading to recurrent cystitis.

3. CVA - anatomy a) Where is the lesion? [primary motor cortex] What happened? b) MRI sagittal - what are the areas pointed to? caudate nucleus, anterior horn of the interior capsule, posterior corpus callosum c) MRI coronal - territories of ACA/MCA/PCA. What other areas are supplied by the MCA? d) Immediate/lifethreatening complications? How do they present? Later complications? e)? PCA - what would happen if you get an occlusion of this artery? Hemianopia
3) MI with coronary artery anatomy What are the weakest points of the myocardium for ventricular rupture to occur? Station 1: Surfer dumped into sand - injured neck (specifically c6 fracture) How would you protect the neck? Axial CT of spine -> what level of the neck is this CT taken at? what lesion can you see? what anatomical features are there? Sagittal MRI of the spine -> what injury is present? what complications may result from this lesion (specifically motor and sensory deficits)? What short and long term complications may arise from this injury (bowel/urinary incontinence)

Anatomy Station

Case of a person with past history of hypertension and previous myocardial infarct. Now presents with focal neurological signs and Atrial fibrillation. Neuro signs included left spastic hemiparesis of arm and leg, and CN 7 affected, and upgoing plantar reflex. Neuroanatomy asked to identify different parts of brain on CT scans. Asked about blood vessel supply to functional areas of the brain, and to identify these areas on CT scan. Asked about the proposed mechanism of death in the person in the case. 1. ANATOMY Story: Case protocol 34 [CEREBROVASCULAR DISEASE] a) What is the lesion that caused this neurological deficit and what areas are likely to be affected? b) How might this pathological process have occurred? c) Image of a CT - transverse section -- name these parts. What is the function of this part? (caudate) d) Image of a CT - coronal section -- which artery supplies these areas? What function is affected if this area was affected (occipital lobe)? What functional areas does the middle cerebral artery supply? Case history 55 year old OBESE businessMAN comes into ED presenting with 2 hour history of CHEST PAIN. Central, retrosternal. BMI 35. 10 year history of HYPERTENSION. 1. What are the cardiac disease risk factors for this guy? (the things in bold, include being a MAN as a factor) 2. What are your differential diagnoses and what do you think is most likely? 3. What investigations can you do NOW to determine youre correct? 4. Pathology of events leading up to MI (or atherosclerosis? Sorry I forgot which.) Coronary vessel anatomy Please take us through the coronary anatomy (theres a model of a heart). Where do the vessels arise? (Above the aortic valve lol) Which vessel supplies SA node? Which supplies AV node? There were 2 questions here on something about whats the most likely to rupture Im sorry I wasnt really sure what they were trying to ask me.

Station 1: Middle aged gentlemen with sudden onset crushing retrosternal chest pain. Previous week had experienced transient chest pain on exertion resolving with rest. Long-standing hypertension and obese. SOB, tachycardic, tachypnoea, hypertensive. Q: What are your differentials?

Q: What are his risk factors for CVD? Q: What investigations would you conduct immediately? Q: Explain briefly pathophysiology of AMI? Q: Explain anatomy of coronary arteries (was given plastic model of heart)? Q: Which coronary arteries supply SA node and AV node? Q: Which arteries drain into coronary sinuses? Q: What are the complications of AMI? Q: Point out at which anatomical locations would myocardial rupture occur and what are the consequences of these ruptures?

Rural 2010 Biomed Questions

WAGGA Pathology: stroke Anatomy: pancreatic (CT scan and XR) Micro: meningitis - causes/identification/treatment Remedial micro: pneumonia Pharm: anti-PLT, anti-coag, etc.

- ibd - Gynae anatmy with reference to the risk or ureteric damage for anat - liver failure - antibiotics yuck but apparently it was the COPD question atypical bacteria - they said it was ok

Path: IBD Anatomy: cervix ulcer and hysterectomy Micro: obstructive LFTs and portal hypertension picture Pharm: pneumonia and RX

arterial, venous and lymphatic drainage of the cervix

Tessa -

Pancreatitis Lymphatics Antibiotic

anyway from what i remember - anat - cardiac and brain tumour, pharm warfarin and h pylori treatment, white book - renal/abg stuff, and path - IBD.

That's what I heard too - apparently for micro a patient presented with pneumonia that turned out to be mycoplasma and her LMO had put her on amoxicillin. Then they had to explain why amoxicillin wouldn't work on mycoplasma and what ABx would be more effective.

Then for the cervical CA one, it was a woman who had an ulcer, noticed on speculum exam. They must have gone into treatment then cos it ended up as complications from the surgery (i.e. local structures) - I'm guessing this is the one about the ureter.

71 yr

old woman with t score -2.9 what are the causes of secondary osteoporosis Whats the difference between t and z score. When would you use a z-score instead of a t-score DEXA ranges for osteoporosis Treatment options What are the ways you give vitamin D and why

38yr old female presents with hard nontender lump DDx for breast lump Whats the triple assessment When do you use US vs mammogram Does fibrocystic change predispose to carcinoma What histological results would make you concerned that fibrocystic change was increased risk of developing into carcinoma (Atypical hyperplasia)

24yr old male had been exercising. Also recently unwell with some URTI. He had polyuria, polydipsia, ketotic fetor, increasingly confused. U/A results showed ketones and glucose Whats your Px Explain the pathophysiology What is their polyuria/polydipsia

What precipitated it Whats the underlying disease (T1DM) ABG analysis Metabolic acidosis with respiratory compensation Explain whats happening to his bicarbonate stores

50 yr old male presents after new onset seizure (twitching of the face and upper left limb) + headaches + sensory changes + decreased memory and mood changes Px and DDx MRI label structures Only had to label lateral ventricles, corpus callosum, mass lesion and surrounding edema. On the model of the brain identify where the lesion is And how this is causing the symptoms Where is personality and memory contained What type of seizure

PHARM

30-40F presents with 12 days of low grade fever, cough productive of a little sputum, malaise and lethargy. Crackles over all lung fields on auscultation. HR 120 BP140/90 RR 26 Temp 37.8. She was seen by her GP, given Amoxicillin and sent home. 1. What is your provisional and differential diagnosis and why? 2. How would you investigate this? The Sputum culture came back positive for mycoplasma pneumoniae. 3. How does Amoxicillin work? 4. Would you keep her on Amoxicillin? What would you prescribe instead? Extra questions: how would you assess whether she should be treated in hospital or at home? -----------ANAT 46F presents for a routine Pap smear. She has been experiencing intermenstrual and post-coital bleeding. On speculum exam she has an ulcerated lesion on the cervix. Rest of the vaginal exam appears normal. 1. What is your differential and provisional dx?

2. How would you investigate this lady? [They were primarily interested in imaging esp transvaginal u/s] 3. What are the anatomical relations of the cervix? [they wanted to know about arterial supply, lymphatic drainage ext and int iliac nodes and sacral nodes and the ligaments supporting the uterus round, broad and uterosacral ligaments] 4. What structures do you need to be wary of during a hysterectomy? (they wanted to know about the relation of the uterine artery to the ureter) ----------MICRO 1. Lab visit (tell us about the gram stain from the patient swab taking microscopy) my patient had a skin lesion with staph aureus they asked what staph aureus looked like on microscopy compared to strep. Then as an extra question they asked what E.Coli looked like on microscopy. The history was the same one from the case protocol on alcoholic liver disease... and we had to interpret the LFTs [ALT,AST and alk phos were normal; Bilirubin, GGT and Globulins were raised; Albumin was low] 2. What is your provisional diagnosis? 3. How would you interpret these LFTs? [Why are the Globulins up?] 4. What is the pathophys behind the clinical manifestations (splenomegaly, ascites, peripheral oedema, bleeding haemorrhoids)? Other questions: Why did she decompensate? [[Eg. Infection, SBP, a binge] What are the other clinical manifestations are there of liver disease? ------------PATH 1. Talk about your cut up/autopsy 26F presents with 6 weeks of bloody and mucous diarrhoea. ?Had a previous episode a few yrs ago. 2. What is your provisional and differential dx? 3. What more would you like to find out on the history and examination? 4. How can you compare between UC and Crohns (on colonoscopy and biopsy)? Extra questions: What are the extraintestinal manifestations? What are the complications of IBD? (Esp mention CRC and drug side effects)

Ours: Anat 1- pneumothorax, given cross section of lungs, label structures, abnormalities, complications Anat resit- spinal cord c7 lesion, effects, complications, imaging Micro- reflux nephropathy, CRF + whitebook Path- lumbar spine, met of prostate cancer + whitebook Pharm- cardiovascular risk factors, HTN treatment, angina and post MI treatment (both pharm and non-pharm measures) Others: Breast cancer and anatomy Cervical cancer and anatomy Frontal lobe tumour and anatomy Pharm of UC Pharm of osteoporosis Path of colon cancer Micro of atypical pneumonia

Station 1 Diagnostics (Hazel Mitchell & Mariott) They choose a lab Ix and you just blabber on about it, sometimes they will stop you to ask specific questions. If you have Hazell Mitchell and Mariott H.Mitchell is the good cop she nods and smiles reassuringly and Mariott picks on little things and asks the specific questions. Case: 22yo female headache, fever, neck stiffness and rash.(pretty much meningitis) What is your PDx? What investigations would you do? What are the main causes of meningitis? What treatment would you start her on? Station 2: Pathology (Sandy McColl and Local) Asked a cut up they can be nice and say tell us something interesting which you saw (which they asked the girl before me) but they asked me something specific I see youve done a couple of skin biopsieswhen would you do a punch and when would you do an ellipse?

Case was about the lady with right hemiplegia (leg, arm, face). She had loss of vision which resolved a couple of days ago (TIA amaurosis fugax!). Your PDx? Given an MRI showed L MCA infarct. What is the pathophysiology behind this? Where can the lesion originate from? What are the causes of death in the first 2 weeks? Station 3: Pharm (Some Sydney pharm dude + Local) Lady 3 weeks post partum, pleuritic chest pain, suspected PE. What imaging? Rx? Monitoring? Station 4: Anatomy (Vu and a Local) Middle aged guy alcoholic intermittent epigastric pain loose stools weight loss (acute on chronic pancreatitis) PDx? Why are you so sure its chronic pancreatitis? Why the stools and weight loss? Showed xray of calcifications pathogenesis Abdo CT (level of kidneys): AA, IVC, descending colon, portal V

The people who had to resit / failed, it was due to Micro station. So any tips brush up on your microthey thought they did ok in it but apparently it wasnt good enough (and Hazel Mitchell just nods and smileseven if you are wrong i.e. smiling assassin). Umm last year Wagga had Pelvic anatomy but some Syd kids apparently had laryngeal anat too (yikes). Resit Micro pneumonia.

They got asked: PDx, DDx, Ix, how to tell if adequate sputum culture, what would u see under LM if sputum / saliva, common MOs causing pneumonia in immunocompetent and immunocompromised host, Rx.

GOOD LUCK GOOD LUCK AND GOOD LUCK!!! Angelina

Вам также может понравиться